You are on page 1of 107

Revise ENTIRE PRELIMS 2024 Current

Affairs through 1500+ MCQs!


Unique Course to Master Current Affairs &
Improve your confidence Levels!

150+ MCQs Every Month (With Solutions)


Apr-23 to April 2024

Offer Price Rs 49

Click Here to Join Now

See Below Sample


July 2023 Current Affairs MCQs
(Solved)

1) Which of the following sectors are


part of the Index of Eight Core
Industries (ICI)?

1. Coal
2. Gold
3. Electricity
4. Textiles
5. Fertilizers

om
Choose the correct code

.c
• a. 1 and 3 only
• b. 2, 4 and 5 only • These 8 core industries accounts for 40.27


c.
d.
1, 3 and 5 only
1, 2, 3, 4 and 5
ap % of the items included in the Index of
Industrial Production (IIP).
m
• Cement production, with a
Answer : c weightage of 5.37 %, saw
m

Index of Eight Core Industries (ICI) the highest growth rate of 15.5
Recently MINISTRY OF COMMERCE
%.
xa

& INDUSTRY released the The


• The production of
combined Index of Eight Core Industries
coal, Petroleum Refinery
(ICI) which increased by 4.3 per cent
.e

(provisional) in May 2023 as compared Products, Fertilizers, Steel and


to the Index of May 2022 Cement increased while Crude
w

Oil, Natural Gas and Electricity


• The Index of Eight Core declined.
w

Industries (ICI), which measures • The Electricity has most


weightage while the Fertilizer
W

the combined and individual


performance of 8 key sectors in has least weightage in the ICI.
the Indian economy.
• ICI is released by the Office of 8 core Weightage and
the Economic Adviser (OEA), industries status
Coal • Weightage –
Department of Industrial Policy
10.33% of ICI.
and Promotion (DIPP), Ministry • Increased by
of Commerce and Industry. 7.2% in May
• The ICI showed a growth of 4.3 2023 over May
% in May 2023 compared to May 2022.
2022.
Crude Oil • Weightage –
8.98% of ICI.

Www.iasbio.com | Www.exammap.com | WhatsApp for UPSC NOTES +918987187161


• Declined by 2) Consider the following statements
1.9% in May with respect to Giant Metrewave Radio
2023 over May Telescope (GMRT)
2022.
1. It is an array of 30 antennas
Natural Gas • Weightage – positioned in a ‘Y’ fashion,
6.88% of ICI.
centred at Pune, Maharashtra.
• Declined by
0.3% in May, 2. It is the first telescope to use X-
2023 over May, Rays to detect the presence of
2022. muons in outer space.

om
Petroleum • Weightage – How many of the statements given
Refinery 28.04% of ICI. above are correct?

.c
Products Increased by
2.8% in May • a. 1 only
2023 over May
2022.
ap •

b.
c.
2 only
Both 1 and 2
• d. Neither 1 nor 2
m
Fertilizers • Weightage –
2.63% of ICI.

Answer : a
m

Increased by
9.7% in May Giant Metrewave Radio Telescope
2023 over May (GMRT)
xa

2022. In recent times uGMRT data are being


used for the ongoing international
.e

Steel • Weightage – efforts on gravitational wave astronomy


17.92 % of ICI.
w

• Increased by • Giant Metrewave Radio


9.2% in May, Telescope (GMRT) is an array of
w

2023 over May, 30 antennas positioned in a ‘Y’


2022. fashion, centred at Pune,
W

Maharashtra.
Cement • Weightage –
• It is used to detect the cosmic
10.33% of ICI.
• Increased by radio waves and support
15.5% in May, interferometry as well as array
2023 over May, mode of operations.
2022. • Atomic hydrogen gas content of
galaxies 9 billion years ago was
Electricity • Weightage – measured using GMRT.
19.85% of ICI. • It was one of the world's six large
• Declined by
0.3% in May,
telescopes that played a key role
2023 over May, in finding the first direct evidence
2022. for the relentless vibrations of the
fabric of the universe.

Www.iasbio.com | Www.exammap.com | WhatsApp for UPSC NOTES +918987187161


• uGMRT is the upgraded version about all the creditors and
of GMRT. borrowers in the financial
• Upgraded GMRT provides system.
wideband frequency coverage • CICs maintain credit information
with improved dynamic range. of borrowers (including
individuals, corporate, SMEs)
3) Consider the following statements which can be accessed by banks
with respect to Credit Information and other lending institutions.
Companies (CICs) • CICs collects public data, credit
transactions & payment

om
1. CICs are companies registered histories of both individuals and
under Credit Information companies.
Companies Act, 2005. • Both banks & Non-banking

.c
2. CICs collects public data, credit financial institutions refer to the
transactions & payment histories CIC’s report before granting
of both individuals and
companies.
ap•
loan or credit card.
Unlike banks, people don’t have
3. Both banks & Non-banking direct access to CICs’ database.
m
financial institutions refer to the • They rate the borrowers in a
CIC’s report before granting loan
m

scale of 300-900 with 900 being


or credit card. the highest rating.
xa

• The companies often need to


How many of the statements given update the information so that the
above are correct?
default player if subsequently
.e

• a. Only one make the repayment their scale


w

• b. Only two should be increased accordingly.


• Banks and finance companies
• c. All Three
w

• d. None normally take decision on the


rating given by the credit
W

Answer : b bureaus.
Credit Information Companies • If the borrower has a rating
(CICs) of over 800, he has a good
The RBI issued penalty of Rs. 1 crore on chance to get a loan or credit
4 CICs for their failure to update credit card easily and at a lower interest
information of borrowers. rate.
• If a borrower defaults, his rating
• “Credit Information Company” will come down. When the rating
means a company formed and declines below 500, chances of
registered under the Companies getting low interest rates and
Act, 1956. even loans or cards also decline
• CICs otherwise called as Credit accordingly.
Information Bureaus have details

Www.iasbio.com | Www.exammap.com | WhatsApp for UPSC NOTES +918987187161


• They levy charges on enquiries • It has sickened and killed an
even if they have no data on the unknown number of sea lions and
entity. dolphins.
• Banks mandate the usage of • Sea lions are Sentinel species,
credit information report (CIR) in animals that can help identify
their credit appraisal process. environmental risks to humans.

4) Consider the following statements Algal Blooms


with respect to Red Tide
• An algal bloom or algae bloom is

om
1. They refer to the Harmful Algal a rapid increase or accumulation
Blooms (HABs) on water bodies. in the population of algae in
2. HABs grow both in fresh and sea freshwater or marine water

.c
water. systems.
3. HABs are mostly found in the • Most blooms are
Tampa Bay area of Florida, USA. ap beneficial because the tiny plants
are food for animals in the ocean.
How many of the statements given • They are the major source of
m
above are correct? energy that fuels the ocean food
web.
m

• a. Only one • Not all algal blooms are


• b. Only two
xa

harmful.
• c. All Three • HABs also include blooms
• d. None of non-toxic species that have
.e

harmful effects on marine


Answer : c ecosystems.
w

Red Tide • The human illnesses caused by


In recent times the red tides is on the rise
w

HABs, though rare, can be


along the Florida’s Gulf Coast of USA
debilitating or even fatal.
killing marine species
W

• HABs affect not only the health


• Red Tide is Harmful algal of people and marine ecosystems,
blooms (HABs) which are red in but also the 'health' of local and
colour. regional economies.
• It is caused by microscopic algae
5) Diversity for Restoration (D4R),
that produce toxins that kill fish
often seen in the news, is associated
and make shellfish dangerous to
with which of the following?
eat.
• The toxins may also make the • a. Initiative of Human Rights
surrounding air difficult to to restore people displaced by
breathe. Ukraine-Russia war
• Humans are unaffected unless • b. Programme by United
they eat infected shellfish. Nations for restoration of

Www.iasbio.com | Www.exammap.com | WhatsApp for UPSC NOTES +918987187161


mangroves along the coasts in • The tool helps the user
Pacific Countries in identifying species that match
• c. It is a tool that aids in their restoration objectives.
agroforestry and systematic • The tool has information
restoration of the ecosystem about 100 plant functional
• d. It is the initiative of the traits that have been considered
Ministry of Social Justice and to offer the best possible solution.
Empowerment to provide • The tool informs the user about
skilled employment to tribal the tree species that offers
people timber, fruit, manure or

om
other commercial benefits.
Answer : c • It also informs about the tree’s
Diversity for Restoration (D4R) status of resilience to

.c
Recently Ashoka Trust for Research in physiological stress such as
Ecology and the Environment (ATREE), extreme high or low
with the help of Bioversity International,
modified Diversity for Restoration
ap temperatures, salinity or acidity
tolerance in the soil.
(D4R) to promote restoration
m
programmes in India 6) Consider the following statements
m

with respect to National Sickle Cell


• Diversity for Restoration (D4R) Anaemia Elimination Mission
is a tool that enables appropriate
xa

agroforestry and aids systematic 1. The mission aims to eliminate


ecosystem restoration. Sickle Cell Anaemia by 2050.
.e

• Diversity for Restoration (D4R) 2. The mission will cover the entire
tool was devised by Bioversity population from 0 to 18 years.
w

International. 3. Sickle cell disease is an inherited


• It was later modified by Ashoka
w

group of blood disorders that is


Trust for Research in Ecology genetic in nature.
W

and the Environment (ATREE) to


adapt it to the Indian context. How many of the statements given
• The tool will help improve the above are correct?
effectiveness of restoration
programmes by providing • a. Only one
manifold benefits to interested • b. Only two
stakeholders while promoting • c. All Three
sustainable development. • d. None
• The online tool precisely aims to
help better decision-making and Answer : b
bring the best outcome for those National Sickle Cell Anaemia
plantation programmes. Elimination Mission
Recently Prime Minster Narendra Modi
launched National Sickle Cell Anaemia

Www.iasbio.com | Www.exammap.com | WhatsApp for UPSC NOTES +918987187161


Elimination Mission to eliminate it by Sickle cell disease (SCD)
2047
• Sickle cell disease is an inherited
• National Sickle Cell Anaemia group of blood disorders that is
Elimination Program was genetic in nature.
introduced in the Union Budget • Red blood cells contain
2023. hemoglobin, a protein that carries
• Mission: To improve care of all oxygen.
Sickle Cell Disease patients for • Sickle cell disease (SCD) is
their better future and to lower a chronic single gene

om
the prevalence of the disease disorder causing a debilitating
through multi-faced coordinated systemic syndrome characterized
approach towards screening and by

.c
awareness strategies o Chronic anaemia,
• Objectives of the Mission: o Acute painful episodes,
o Provision of affordable
and accessible care to all
ap o Organ infarction and
chronic organ damage and
SCD patients, o Significant reduction in
m
o To ensure quality of care life expectancy.
m

for SCD patients and • SCD is a genetic condition that


o To reduce the prevalence is present at birth.
xa

of SCD. • It is inherited when a child


• It focuses on eliminating sickle receives two genes—one from
cell disease, particularly among each parent—that code
.e

tribal populations of the country. for abnormal hemoglobin.


• It is part of the National Health
w

Mission (NHM), aims 7) Consider the following statements


w

to eliminate sickle cell genetic with respect to Euclid telescope


transmission by the year 2047.
W

• Over a period of three years, 1. It is being used in Venus


spanning from 2023-24 to 2025- exploration to study the
26, the program targets screening atmosphere of the Venus.
approximately 7.0 crore people. 2. The telescope was launched by
• It is implemented in 17 high- NASA.
focus states across the country.
• Platforms such as Which of the following statement(s)
monthly Village Health is/are correct?
Sanitation and Nutrition
• a. 1 only
Committee (VHSNC) shall be
• b. 2 only
leveraged to make awareness
• c. Both 1 and 2
about the disease.
• d. Neither 1 nor 2

Www.iasbio.com | Www.exammap.com | WhatsApp for UPSC NOTES +918987187161


Answer : d 3. In Brij Bhushan vs the State of
Euclid Space Telescope Delhi case, the Supreme Court
Euclid Space Telescope was recently held that pre-censorship on
launched by European Space Agency to freedom of speech is
survey billions of galaxies unconstitutional.

• The Euclid mission hopes to How many of the statements given


understand the evolution of the above are correct?
Universe by looking at the light
emitted from galaxies 10 billion • a. Only one

om
years ago. • b. Only two
• It was launched by European • c. All Three
Space Agency from Cape • d. None

.c
Canaveral in Florida.
• Euclid is a cosmology survey Answer : b
mission, optimised to determine
the properties of dark energy and
ap
Online Free Speech
Recently Karnataka High Court
dark matter on universal scales. dismissed Twitter’s challenge to the
m
• The telescope will also focus on issuance of blocking orders by the
gleaning more information on Ministry of Electronics and Information
m

dark energy and dark matter. Technology (MeitY)


xa

• Dark energy is the undetectable


• The judgment undermines the
form of energy that is causing the
right to free speech and
expansion of space.
.e

expression.
• About 68% of the universe is
• The judgment also paves the way
made of dark energy while dark
w

for the state to exercise


matter makes up 27%. Only 5%
unchecked power while taking
w

of the Universe is made of


down content without following
“normal matter”.
W

established procedure.
• Once the telescope is operational,
• Section 69A of the Information
it will scan more than 1/3rd of
Technology Act, 2000 empowers
the sky.
the state to issue blocking orders
8) Consider the following statements: in cases of emergency on the
grounds such as
1. Section 69 of the IT Act allows o Sovereignty and integrity
the government to issue content- of India,
blocking orders to online o Defence of India,
intermediaries. o Security of the State and
2. Free speech can be restricted o Friendly relations with
under Article 19(2) and Section foreign States.
69A of IT Act, 2000 on the • Section 69 of the IT Act allows
grounds of fake news. the government to issue content-

Www.iasbio.com | Www.exammap.com | WhatsApp for UPSC NOTES +918987187161


blocking orders to online Answer : a
intermediaries. Energy Transition Index (ETI) 2023
• Misinformation and fake Recently World Economic Forum
news are not grounds under released Energy Transition Index 2023
which free speech can be as part of their annual exercise
restricted under Article 19(2) and
Section 69A of Information • India ranked 67th on Energy
Technology Act, 2000. Transition Index which is topped
• In Brij Bhushan vs The State of by Sweden.
Delhi case the Supreme Court • The index is released by World

om
held that pre-censorship Economic
on freedom of speech is Forum in collaboration with Acc
unconstitutional. enture.

.c
• Shreya Singhal vs Union of • India it is the only major
India, the Supreme Court held economy with energy transition
that blocking under Section 69A
must conform to the grounds
ap momentum accelerating across
all dimensions.
of India’s national security,
m
sovereignty, or public order. Key findings of the index
m

• Besides India, Singapore is the


9) Consider the following statements
only other major economy
xa

with respect to Energy Transition Index


2023 showing "true momentum by
advancing sustainability, energy
.e

1. The index is released by the security and equity in a balanced


International Energy Agency way.
w

(IEA). • Among the world’s 10 largest


economies, only France features
w

2. For the first time, India ranked in


the top 10 of the Energy in the top 10.
W

transition index. • Sweden is followed by Denmark,


3. India it is the only major Norway, Finland and
economy with energy transition Switzerland in the top five on the
momentum accelerating across list of 120 countries.
all dimensions. • Even as the global energy
transition is plateauing due to
How many of the statements given equity challenges, major
above are correct? economies are showing
significant progress.
• a. Only one • Global average ETI
• b. Only two scores increased by 10% since
• c. All Three 2014, but showed only marginal
• d. None growth in the past three years.

Www.iasbio.com | Www.exammap.com | WhatsApp for UPSC NOTES +918987187161


• The top 10 countries account for • Facilitating research in
only 2% of global CO2 emissions universities is expected to be one
from fuel combustion and 4% of of the key focus areas for NRF.
total energy supply. • It will focus on cultivating the
culture of research in universities
10) Consider the following statements and colleges.
with respect to National Research • It will also focus on funding of
Foundation (NRF) projects in peripheral, rural and
semi-urban areas.
1. NRF is an apex body to promote, • The NRF is estimated to have an

om
fund and mentor scientific initial budget of Rs 50,000 crore
research in higher education over a five-year period between
institutions across India. 2023 and 2028.

.c
2. The Prime Minister is the ex- • India barely spends 0.7 % of its
officio president of the NRF GDP on research and
board.
3. NRF will be funded and aided by
ap•
development.
The gross expenditure on
the Science and Engineering
m
R&D declined from 0.84 per
Research Board (SERB). cent in 2008 to about 0.69 per
m

cent in 2018.
How many of the statements given • The Science and Engineering
above are correct?
xa

Research Board (SERB) will be


subsumed in the NRF.
• a. Only one
• About 65% of funds from SERB
.e

• b. Only two
• c. All Three had been cornered by the IITs,
w

• d. None and only 11% flowed to projects


in state universities.
w

• The NRF would also promote


Answer : b
research in social sciences, arts
W

National Research Foundation (NRF)


Recently the union government decided and humanities, with one of the
to establish NRF to address the shortage primary aims being finding
in the field of Research and solutions to the big problems
Development (R&D) and increase the facing Indian society.
funding to it • The NRF would
be administratively under
• National Research Foundation the Department of Science and
(NRF) as an apex Technology (DST).
body to promote, fund and • NRF would also provide
mentor scientific research in fellowships for post-doctoral
higher education research, the funding for which
institutions across the country. is not available right now.

Www.iasbio.com | Www.exammap.com | WhatsApp for UPSC NOTES +918987187161


• The NRF’s functioning will • d. Neither 1 nor 2
be governed by an executive
council chaired by the Principal Answer : d
Scientific Advisor to the Colombo Security Conclave (CSC)
government of India. Under the framework of CSC scientists
from Bangladesh and Mauritius have
NRF governing board recently used India’s research vessel
‘Sagar Nidhi’ for joint ocean expedition
• The Prime Minister will be in Indian Ocean
the ex-officio president of the

om
board. • Colombo Security Conclave
• The Minister of Science and (CSC) is a mini-lateral group in
Technology and the Minister of the Indian Ocean region.

.c
Education will be the ex-officio The members of CSC
vice presidents. includes Sri Lanka, Maldives,
• It will be governed by 16
member which includes
ap

Mauritius and India.
The observers of CSC
o 2 members from DST, includes Bangladesh and the
m
o 5 from industry, Seychelles.
o 1 from humanities and • The task of
m

o 6 experts who would be CSC includes maritime security,


countering terrorism, and
xa

selected depending on the


nature of the project being cybersecurity.
evaluated. • It provides an opportunity for
.e

India to address its own strategic


11) Consider the following statements concerns in the Indian Ocean.
w

with respect to Colombo Security • Recently scientists from


Conclave (CSC) Bangladesh and
w

Mauritius used the Indian


W

1. It is a mini-lateral group in the research vessel Sagar Nidhi for


Indian Ocean region with India, ocean expedition under the
Sri Lanka, Maldives and framework of CSC.
Australia as members. • The scientists will during the
2. It is an exclusive economic collaboration undertake research
grouping that focuses to improve on the ocean data to predict and
regional trade activities. manage changes in the marine
environment and variation in
Which of the statement(s) given above ocean parameters.
is/are correct? • The cruise is conducted by
Indian National Centre for
• a. 1 only Ocean Information Services
• b. 2 only (INCOIS) under the Ministry of
• c. Both 1 and 2 Earth Sciences.

Www.iasbio.com | Www.exammap.com | WhatsApp for UPSC NOTES +918987187161


12) Consider the following pairs related • a. Temperate cyclons of North
to Buddha’s teachings America
Buddha’s • b. Heatwaves of Europe
teachings Meaning • c. Diamond reserves of Africa
• d. Warm local winds of Japan
1. Pragya – Wis
dom Answer : b
2. Sheel – Co Heatwaves
ncentration Yago and Zoe are recent heatwaves in
3. Samadhi – Ethi Seville, Spain of Europe

om
cal Conduct
• Seville is among the first cities in
How many of the statements given the world to name and rank

.c
above are correct? heatwaves.
• Seville’s system to rank


a.
b.
Only one
Only two
ap heatwaves are based on criteria
such as humidity, impacts on
• c. All Three human health, and daytime and
m
• d. None night time temperatures.
• India has no such naming
m

Answer : a system for heatwaves, although it


Recently on the occasion of Dharma
xa

does suffer massively from


Chakra Pravartana Divas the President heatwaves.
of India called upon the youth to draw • The countries such as Greece,
.e

inspiration from the teachings of Chile and United States has


Buddha adopted a law to establish a heat
w

wave ranking system.


• The Buddha’s teachings
w

• The World Meteorological


were transmitted orally by his Organization (WMO)
W

disciples during his lifetime he has recognized the interest in


established the Buddhist developing heatwave ranking and
monastic order (sangha). naming systems.
• Sheel means Ethical Conduct or • Naming could help in mitigate
virtue. the disaster faster and could clear
• Samadhi means concentration. the air during the
• Pragya means wisdom. communication.
• In Buddhism, samadhi is the last
of the eight steps leading to 14) Consider the following statements
enlightenment (the Eightfold with respect to Paris Club
Path).
1. It is an informal group that
13) Yago and Zoe, sometimes seen in supports the expansion of
the news recently, refers to?

Www.iasbio.com | Www.exammap.com | WhatsApp for UPSC NOTES +918987187161


permanent seats in the UN
Security Council (UNSC)
2. The Club has 22 members
including India and Japan.

Which of the statement(s) given above


is/are correct?

• a. 1 only
• b. 2 only

om
• c. Both 1 and 2
• d. Neither 1 nor 2

.c
Answer : d
Paris Club
Recently China did not join common
creditor platform created by Paris Club
ap
to resolve Sri Lanka’s debt crisis and
m
also said that china will support Sri
Lanka bilaterally
m

• The Paris Club, started in 1956 is


xa

an informal group that


helps debtor countries to
.e

manage their debt problems.


• The Club has 22 permanent
w

members excluding India.


w
W

• Recently “official creditor


committee”, co-chaired by India,
Japan, and France, to discuss Sri
Lanka’s request for debt
treatment was opted out by
China.
• Other creditor countries can
participate in negotiation
meetings on a case by case basis,
provided that certain conditions
are met.
• Its work is based on a number
of rules and principles agreed by

Www.iasbio.com | Www.exammap.com | WhatsApp for UPSC NOTES +918987187161


creditor countries, which • It was given Geographical
facilitates the decision making Indication (GI) tag in 2019.
process and the conclusion of
agreements. 16) Consider the following statements
with respect to Liberalised Remittance
15) Consider the following statements Scheme (LRS)
with respect to Thazhapayya
1. Indian citizens may use the
1. It is a traditional handwoven Liberalized Remittance Scheme
screw pine mat which is known (LRS) to send money abroad.

om
for their healing properties. 2. An individual can send up to
2. Thazhava village in Tamil Nadu $250,000 to an overseas
is famous for making jurisdiction per year under the

.c
Thazhapayya. Liberalised Remittance Scheme
3. It is losing its originality since it (LRS).
has no Geographical Indication
(GI) tag.
ap
3. Corporations, partnership
businesses, Hindu Undivided
Families (HUF), trusts, etc.
m
How many of the statements given cannot use it.
above are correct?
m

How many of the statements given


• a. Only one above are correct?
xa

• b. Only two
• c. All Three • a. Only one
.e

• d. None • b. Only two


• c. All Three
w

Answer : a • d. None
Thazhapayya
w

In recent times Thazhapayya is facing Answer : c


W

extinction due to lack of patronage Liberalised Remittance Scheme


Recently RBI increased the threshold to
• Thazhapayya is a traditional 2, 50,000 for Liberalised Remittance
handwoven screw pine mats Scheme (LRS)
known for its healing properties.
• It is made of the leaves of the • Liberalised Remittance Scheme
thazha plant, which is also called allows all resident individuals,
kaitha or attukaitha in local including minors to freely remit
parlance. up to USD 2, 50,000 per
• A village named Thazhava near financial year for both current
Karunagappally in Kollam and capital account transaction.
district of Kerala is famous for • The Scheme was introduced on
thazhappaya making. February 2004, with a limit of
• They are made mostly in Kerala. USD 25,000.

Www.iasbio.com | Www.exammap.com | WhatsApp for UPSC NOTES +918987187161


• The Scheme is not available to 17) Consider the following statements
corporates, partnership firms, with respect to Chidambaram Nataraja
HUF, Trusts etc. Temple
• The Prohibited Items under the
Scheme are: 1. The temple was constructed
o Remittance for trading in during the 10th Century when
foreign exchange abroad. Chidambaram used to be the
o Gifting by a resident to capital of the Chola dynasty.
another resident, in foreign 2. A unique feature of this temple is
currency, for the credit of the gold image of Lord Nataraja.

om
the latter’s foreign 3. The temple is built in the
currency account held Dravidian style of temple
abroad under LRS. architecture.

.c
o Remittance for any
purpose specifically How many of the statements given
prohibited under Schedule-
I or any item restricted
ap
above are correct?

• a. Only one
m
under Schedule II of
• b. Only two
Foreign Exchange
• c. All Three
m

Management (Current
• d. None
Account Transactions)
xa

Rules, 2000. Answer : c


▪ Schedule-I includes
Chidambaram Sri Sabhanayagar
purchase of lottery
.e

Temple
tickets/sweep Recently there were controversies
w

stakes, proscribed between The Podhu Dikshitars and


magazines, etc. HR&CE officials over the
w

o Remittance from India for administration of the temple


margins or margin calls to
W

overseas exchanges / • The ancient Sabhanayagar


overseas counterparty. temple, also known as the
o Remittances for purchase Nataraja temple, is in
of foreign currency Chidambaram in the Cuddalore
convertible bonds district of Tamil Nadu.
(FCCBs) issued by Indian • The administration of the temple
companies in the overseas has been a contentious subject
secondary market. between the Podhu Dikshitars
• Transactions facilitated using (administrators-cum-priests) and
international credit cards while the State government.
being overseas would not fall • It enjoyed the patronage of
under the LRS umbrella. different rulers including Simha
Varman of the Pallava dynasty,

Www.iasbio.com | Www.exammap.com | WhatsApp for UPSC NOTES +918987187161


Aditya I of the Chola dynasty, • Quasars are tremendously active
Krishnadevaraya of the supermassive black
Vijayanagara Empire, and holes millions to billions of times
Nayaka kings. more massive than our sun,
• The gold-roofed stage or dancing usually residing at centers of
hall is the sanctum sanctorum of galaxies.
the Chidambaram temple. • The researchers used Quasars as
• The Raja Gopuras present now a "clock" to measure time in the
was built in various periods, early universe.
spanning many centuries which • Researchers found that time

om
are uniform in size and passed more slowly in the early
structure. universe than it does today.
• Every part of the Nataraja image • This finding is consistent with

.c
is directly expressive not of any Einstein's theory of general
superstition or dogma, but of ap relativity.
evident facts. • It is also suggested that time is
• The Chidambaram Temple is not absolute, and that it can be
m
unique since the presiding deity affected by the environment.
worshipped is a metal icon of
m

Lord Nataraja in contrast to 19) Consider the following statements


statues of deities made of stone with respect to Zo People
xa

found in other temples.


• Chidambaram temple is owned 1. The Chins of Myanmar, the
by the Denomination of Podu Mizos of Mizoram and the Kukis
.e

Dikhistars. of Bangladesh are collectively


called the Zo people.
w

18) Quasars, often seen in the news, is 2. The Zomi tribes are an
w

associated with which of the following? indigenous community living


along the frontier of India and
W

• a. New species found in the Burma are a sub-group of the Zo


Western Ghats people.
• b. Supermassive black holes
• c. Antibiotics to cure drug- How many of the statements given
resistant infections above are incorrect?
• d. Substance used in the
manufacture of Coca-Cola • a. 1 only
• b. 2 only
Answer : b • c. Both 1 and 2
Quasars • d. Neither 1 nor 2
Recently the researchers used the
brightness fluctuations of Quasars as a Answer : d
way to measure time Zo People

Www.iasbio.com | Www.exammap.com | WhatsApp for UPSC NOTES +918987187161


Recently Mizoram Chief Minister called politically implausible to carve
for a reunification of the Zo people to out areas from Manipur, Tripura,
restore peace in the violence-torn and Bangladesh to integrate with
Manipur state Mizoram.

• Zo peoples are a group of 20) Consider the following statements


tribes that live in Myanmar, with respect to Mo Jungle Jami Yojana
India and Bangladesh along
with many sub-tribes and 1. The scheme is introduced by the
clans such as the Chin, Kuki, State of Kerala, to recognise the

om
Mizo, Lushei, Zomi, Paitei, forest rights of tribal people.
Hmar, Ralte, Pawi, Lai, Mara, 2. The scheme works in line with
Gangte, Thadou, etc. the Central government’s Forest

.c
• They speak a group of Tibeto- Rights Act, 2006.
Burman languages. 3. Under the scheme, all un-
• The Chin Hills or Indo-Chin hill
ranges are home to a large
ap surveyed, forest and zero area
villages will be converted into
number of tribes that come under revenue villages to provide basic
m
the Zo umbrella. service.
• The tribes are believed to
m

have migrated from Which of the above statements


is/are incorrect?
xa

China through Tibet to settle in


Myanmar.
• Even before the 2021 military • a. Only one
.e

coup in Myanmar, tens of • b. Only two


thousands of Zo migrated to • c. All Three
w

Mizoram, migration across the • d. None


w

porous international border had


been constant and seamless. Answer : a
W

Mo Jungle Jami Yojana


• Apart from ethnicity and religion,
Recently the scheme was announced by
the tribes are knit together
Odisha to address the gaps in the Forest
by Christianity.
Rights Act, 2006 and to strengthen
• The Kuki-Zomi of Manipur share forest rights among tribals and forest
the history and memories of the dwellers
violent Mizo nationalist
movement of the 1960s. • Mo Jungle Jami Yojana is
• Most displaced Zo people who a scheme by Odisha
have arrived in Mizoram from government to strengthen forest
Manipur are now in the districts rights among tribals and forest
of Aizawl, Kolasib and Serchhip. dwellers across the state’s
• Zo reunification movement is (Odisha) districts.
unlikely to succeed because it is

Www.iasbio.com | Www.exammap.com | WhatsApp for UPSC NOTES +918987187161


• The scheme aims to ensure • The rights would be distributed
livelihood and food security for across all Gram Sabhas and
the Scheduled Tribe and forest villages.
dwelling population. • The Odisha state is home to 62
• If implemented, Odisha would different tribes of which 13 are
become the first in India to recognised as Particularly
recognise community forest Vulnerable Tribal Groups
rights along with individual (PVTG).
rights offered by the Centre. • The tribal population in Odisha
• Under the scheme, all un- amounts to 22.85 % of the

om
surveyed, forest and zero area overall population in the state.
villages will be converted into
revenue villages to provide 21) Consider the following statements

.c
basic service such as water with respect to Indian Ocean Geoid Low
supplies, road connectivity, (IOGL)


schools and healthcare.
The scheme is conceived to
ap1. Indian Ocean Geoid Low (IOGL)
is the massive gravity hole in the
m
bridge the gaps and address
critical issues not targeted in the Indian Ocean.
2. The earth does not have uniform
m

central scheme (FRA).


• The central scheme recognises gravity throughout its surface.
xa

individual forest rights, but the 3. The strength of the gravitational


state scheme delivers the same pull at any part of the world is
benefits for community rights dependent on the mass of the
.e

and community forest rights Earth’s crust.


w

(CFR) that is lagging in the


How many of the statements given
existing scheme.
above are incorrect?
w

• The scheme will provide


ownership of land and access to
W

• a. Only one
forest resources to the • b. Only two
beneficiaries according to their • c. All three
entitlement and join them with • d. None of the above
mainstream development
programmes of the government. Answer : d
• According to the scheme all Indian Ocean Geoid Low (IOGL)
eligible claimants, mainly single Recently Researchers from the Indian
women and PVTGs will receive Institute of Science have uncovered the
land titles and record cause of Indian Ocean Geoid Low
corrections will be made for all (IOGL)
title holders.

Www.iasbio.com | Www.exammap.com | WhatsApp for UPSC NOTES +918987187161


• IOGL is located about 1,200 3. CMV hosts include cucumber,
kilometres in the southwest of melon, eggplant, and tomato.
the southernmost tip of India.
• It has such a low pull of gravity How of many the above statements are
that the sea level of the Indian correct?
Ocean over this region is around
106 metres below the global • a. only one
average. • b. Only two
• The remains of the ancient • c. All three
ocean, which is about 965 • d. None of the above

om
kilometers below the Earth's
crust, just under Africa, is Answer : b
thought to be causing the low CMV and ToMV

.c
In recent times the price rise in the crops
gravity around the region.
such as tomato and cucumber are due to
• The molten rock plumes are also
thought to be contributing to low
gravity around the region.
ap
the mosaic viruses (CMV and ToMV)
that affected the crops
m
• The researchers found that the • Cucumber mosaic virus (CMV)
remains of the ancient ocean and and Tomato mosaic virus
m

the molten rock plumes are both (ToMV) are two plant
contributing to the low gravity pathogens have similar names
xa

around the region. and cause similar damage to


crops, but they belong
Earth’s Gravity
.e

to different viral families,


• Earth’s Gravity is Non uniform. and spread differently.
w

• ToMV belongs to
• The strength of the gravitational
the Virgaviridae family and is
w

pull at any part of the world


is dependent on the mass of the closely related to the tobacco
W

Earth’s crust, mantle and mosaic virus (TMV).


• ToMV hosts include tomato,
core beneath that particular
region. tobacco, peppers, and certain
other ornamental plants.
22) Consider the following statements • ToMV spreads mainly
with respect to Cucumber mosaic virus through infected seeds, saplings,
(CMV) and Tomato mosaic virus agricultural tools and often
(ToMV) through the hands of nursery
workers.
1. They are zoonotic viruses that • ToMV can remain dormant in
cause skin disease in humans. weeds and plant remains around
2. ToMV hosts tomato, tobacco, the field and affect later.
peppers, and certain ornamental • CMV has a much larger host
plants. pool that includes cucumber,

Www.iasbio.com | Www.exammap.com | WhatsApp for UPSC NOTES +918987187161


melon, eggplant, tomato, carrot, originates from the digestive
lettuce, celery, cucurbits, etc. system of protected sperm
• CMV is spread by aphids, which whales.
are sap-sucking insects. • Ambergris is a rare substance,
• CMV too can spread through which contributes to its high
human touch, but the chances of demand and high price in the
that are extremely low. international market.
• Controlling CMV is more • The freshly passed ambergris is
difficult, given the large number a light yellowish substance and is
of hosts the virus can live on. fatty but as it ages it turns waxy

om
• Conditions of high temperature and gets red-brownish.
followed by intermittent rain, • Traditionally, it is used
which allow aphids to multiply, to produce perfumes which have

.c
are conducive to the spread of notes of musk.
CMV. • While there is a ban on the
• Both viruses can cause almost
100% crop loss unless properly
ap possession and trade of
ambergris in countries like the
m
treated on time. USA, Australia and India.
• They can be controlled by • In India, sperm whales are a
m

following biosafety standards in protected species under Schedule


nurseries and compulsory seed 2 of the Wildlife Protection Act,
xa

treatment to stop the spread of 1972.


ToMV. • The possession or trade of any of
its by-products, including
.e

23) Ambergris, often seen in the news, Ambergris and its by-products,
is associated with which of the
w

is illegal under provisions of


following? the Wildlife Protection Act, 1972.
w

• a. Golden polymetallic nodule 24) Consider the following statements


W

• b. Waxy substance found in with respect to khazan lands


marine animals
• c. It is the closest nebula to the 1. khazans are coastal wetlands of
Milky Way galaxy Goa.
• d. Indigenous developed 2. Khazans are a unique agro-
vaccine for Covid-19 aquacultural system in Goa.
3. This system of traditional
Answer : b agriculture emerged during
Recently carcass of a sperm whale Portuguese rule in India.
washed up on a shore of the Canary
Island of La Palma, Spain How of the above statements are
correct?
• Ambergris, which means grey
amber, is a waxy substance that • a. Only one

Www.iasbio.com | Www.exammap.com | WhatsApp for UPSC NOTES +918987187161


• b. Only two 2. The scheme is implemented by
• c. All three the Ministry of Micro, Small and
• d. None of the above Medium Enterprises.

Answer : b Which of the statement(s) given above


Goa khazan lands is/are correct?
Recently National Green Tribunal set
aside the order passed by the Goa • a. 1 only
Coastal Zone Management Authority • b. 2 only
(GCZMA) to dismiss the proceedings of • c. Both 1 and 2

om
illegal construction and illegal filing • d. Neither 1 nor 2
activities in eco-sensitive regions of
khazan lands in Goa Answer : c

.c
Gramodyog Vikas Yojna
• khazans are coastal wetlands of Recently Delhi Lieutenant Governor


Goa.
They were reclaimed from
ap
distributed Honey Bee-Boxes and
Toolkits to beneficiaries under
mangrove forests, probably in the 'Gramodyog Vikas Yojna'
m
pre-Christian era by an intricate
system of dykes, sluice gates and • Gramodyog Vikas Yojana
m

canals. (GVY) scheme is for promotion


• They are put to multiple and development of village
xa

productive uses such as industries through common


agriculture, aquaculture and salt facilities, technological
.e

panning. modernization, training and other


• Even before the Portuguese support and services for
w

came to rule Goa, Khazans promotion of village Industries.


system was practised in India. • It is under Ministry of Micro,
w

• The institutions Small and Medium Enterprises.


W

called Gaunkaris (village- • GVY has the following


association) regulated Khazans components/ verticals from the
system. activities under Village
• Under the Portugese rule, Industries:
these Khazans were renamed
as comunidads. 1. Wellness & Cosmetics Industry
(WCI)
25) Consider the following statements 2. Handmade Paper, Leather &
with respect to Gramodyog Vikas Yojna Plastic Industry (HPLPI)
3. Agro Based & Food Processing
1. The scheme aims to develop and Industry (ABFPI)
promote village industries. 4. Mineral Based Industry (MBI)
5. Rural Engineering & New
Technology Industry (RENTI)

Www.iasbio.com | Www.exammap.com | WhatsApp for UPSC NOTES +918987187161


6. Service Industry enlightenment, and it sets out the
basic teachings of Buddhism,
including the Four Noble Truths
26) Consider the following statements and the Noble Eightfold Path.
with respect to Dhamma cakka • The sutta begins with the Buddha
pavattana Sutta describing the two extremes that
should be avoided by those who
1. It refers to the first sermon seek liberation.
preached by Buddha after his • The Four Noble Truths includes:
enlightenment. o The truth of suffering

om
2. It comprises the four noble truths o The truth of the cause of
and the noble eight-fold path. suffering
3. It is also known as the Turning of o The truth of the cessation

.c
Wheels of Dharma. of suffering
o The truth of the path to the
How many of the above statements are
correct?
ap

cessation of suffering
The Noble Eightfold
m
Path includes:
• a. Only one o Right view
m

• b. Only two o Right intention


• c. All Three o Right speech
xa

• d. None o Right action


o Right livelihood
Answer : c o Right effort
.e

Dhamma Cakka- Pavattana Sutta o Right mindfulness


(Pali) or Dharma Cakra Pravartana
w

o Right concentration.
Sutra (Sanskrit)
w

27) Consider the following statements


• It is also known as the Turning with respect to Symbols Order, 1968
W

of Wheels of
Dharma and comprises the Four 1. It empowers Election
Noble Truths and the Noble Commissioner to resolve the
Eight-Fold Path. dispute between two factions
• The setting in motion of the within the recognised political
Wheel of Dhamma, is a metaphor party.
for the spread of the Buddha's 2. The order of election
teachings which cannot be commissioner regarding faction
stopped. dispute is non-binding in nature
• The Setting in Motion of the and can be appealed before High
Wheel of Dhamma is a key courts.
sutta in the Buddhist tradition.
• It is the first discourse that the Which of the following statement(s)
Buddha gave after his is/are correct?

Www.iasbio.com | Www.exammap.com | WhatsApp for UPSC NOTES +918987187161


• a. 1 only and MLAs have supported one of
• b. 2 only the factions.
• c. Both 1 and 2
• d. Neither 1 nor 2 28) Dark Patterns, sometimes seen in the
news recently, refers to?
Answer : a
Election Commission’s power on • a. It is a pattern found in the
faction within parties dark matters
Recently Maharashtra Deputy Chief • b. It is an unfair trade practice
Minister wrote to the Election that manipulates user choices

om
Commission (EC) to seek recognition of • c. It is a pattern witnessed after
his faction as the real Nationalist wild fire in forests
Congress Party (NCP) • d. It is skin pigmentation

.c
disorder in African countries
• Symbols Order, 1968 empowers
Election commissioner to
resolve the dispute between two
ap
Answer : b
Dark Patterns
factions within the party. Recently The Department of Consumer
m
o This applies to disputes Affairs and the Advertising Standards
in recognised national Council of India (ASCI) held a joint
m

and state parties. consultation with stakeholders on the


• The order of Election menace of ‘dark patterns’
xa

commissioner is binding in
nature. • A dark pattern unfair trade
.e

• The election commissioner while practice that is intentionally


deciding the case taking into crafted to manipulate or deceive
w

account all the available facts users into making certain


and circumstances of the case choices or taking specific actions
w

and hearing (their) that may not be in their best


W

representatives of each faction. interest.


• For splits in registered but • It is a deceptive practice
unrecognised parties, the ECI employed to influence user
usually advises the warring behaviour in a way that benefits
factions to resolve their the company especially E-
differences internally or to commerce implementing it.
approach the court. • There are no specific
• Before 1968, the EC issued regulations to regulate dark
notifications and executive orders patterns.
under the Conduct of Election • The dark patters infringe the
Rules, 1961. consumer rights mentioned
• In almost all such disputes so far, under Consumer Protection Act,
a clear majority of party 2019 but there are no provisions
delegates/office bearers, MPs against Dark patterns.

Www.iasbio.com | Www.exammap.com | WhatsApp for UPSC NOTES +918987187161


• Some of the most common Dark 2. Reduce the need for holding
Patterns includes: forex reserves
o Creating a sense of 3. Mitigates the currency risk for
urgency or scarcity while Indian businesses
online shopping. 4. Improves the bargaining power
o Bait and switch of Indian businesses
technique - Advertising
one product or service but How many of the statements given
delivering another, often above are correct?
of lower quality.

om
o Shaming wherein a • a. Only one
consumer is criticised for • b. Only two
not conforming to a • c. Only three

.c
particular belief. • d. All four
o The forced action of
signing up for a service to
access content.
ap
Answer : c
Internationalisation of Rupee
Recently Reserve Bank of India’s (RBI)
m
o Hidden costs where the
inter-departmental group (IDG) on
bill is revised or costs are
Wednesday said that the rupee has the
m

added when the consumer


potential to become an internationalised
is almost certain to currency
xa

purchase the product.


o Disguised advertisements • Internationalisation of Rupee is
of a particular product by
.e

a process that involves


way of depicting it as increasing the use of the rupee
w

news. in cross-border transactions.


• The Advertising Standards • It involves promoting the rupee
w

Council of India (ASCI) was for import and export trade and
established in 1985
W

then other current account


• ASCI is committed to the cause transactions, followed by its use
of Self-Regulation in in capital account transactions.
Advertising, ensuring the • It will require full convertibility
protection of the interests of of the currency on the capital
consumers, while being fair to account and cross-border transfer
the advertising industry. of funds without any restrictions.
• Currently, the US dollar, the
29) Which of the following is/are the
Euro, the Japanese yen and the
benefits of Internationalisation of
pound sterling are the leading
Rupee?
reserve currencies in the world.
• Advantages of
1. Inclusion of rupee in the SDR
basket internationalisation of the
rupee are:

Www.iasbio.com | Www.exammap.com | WhatsApp for UPSC NOTES +918987187161


o Reduced currency risk for 2. The Green credits will be
Indian businesses tradable.
o Reduced need for foreign 3. Central Pollution Control Board
exchange reserves (CPCB) will be the administrator
o Enhanced bargaining of the programme.
power for Indian
businesses How many of the statements given
o Enhanced global stature above are correct?
and respect for India
• Prerequisites for • a. Only one

om
internationalization of the • b. Only two
rupee include: • c. All Three
o Size of the economy and • d. None

.c
the
o volume of international
Answer : b
transactions
o A stable currency
ap
Green Credit Programme (GCP)
Government of India has recently
released the draft rules for Green Credit
m
o A liquid currency
Programme (GCP) which was
• Internationalization of rupee will
announced during 2023-24 budget
m

not directly help in including the


Indian rupee in IMF’s SDR. • GCP aims at incentivising
xa

• A currency included in the SDR voluntary environmental


basket must meet two criteria: actions of various stakeholders.
.e

• It allows green projects to


achieve optimal returns beyond
w

carbon reductions alone.


• The Green Credit Programme
w

serves as a powerful tool


W

for incentivizing sustainable


actions and promoting
sustainable living among
individuals, companies, and local
bodies.
• Individuals, industries, Farmers
producer’s organisations
30) Consider the following statements (FPOs), urban local bodies
with respect to Green Credit Programme (ULB), gram panchayats and
(GCP) private sectors will be able to
earn green credit for
1. It can be earned by individuals,
environment-friendly actions.
farmer-producer organisations
• The environment-friendly
(FPO) and industries.
actions include planting trees,

Www.iasbio.com | Www.exammap.com | WhatsApp for UPSC NOTES +918987187161


conserving water, waste 3. The portal is developed by
management and reducing air Electronics Development
pollution. Corporation of India (EDCI).
• Green credits will be tradable,
creating a potential market How many of the statements given
platform where participants can above are correct?
sell their earned credits.
• Indian Council of Forestry • a. Only one
Research and Education • b. Only two
(ICFRE) will act as the program • c. All Three

om
administrator. • d. None
• The 8 sector which qualify
for Green Credits include: Answer : b

.c
o Tree plantation-based
GatiShakti Sanchar Portal
GatiShakti Sanchar Portal was
green credit ap
launched to speed up approvals for
o Water-based green credit
laying of fibre and tower installations.
o Sustainable agriculture-
m
based green credit • GatiShakti Sanchar portal is
o Waste management-based
a collaborative institutional
m

green credit mechanism between all


o Air pollution reduction-
stakeholders including Central
xa

based green credit and State/UT Government(s),


o Mangrove conservation
Local bodies, and Service
and restoration-based
.e

Providers.
green credit • The portal will centralise and
w

o Eco mark based green


speed up approvals for laying of
credit fibre and tower installations, and
w

o Sustainable building and


has legal backing for the Right of
infrastructure-based green
W

Way (RoW).
credit • A right-of-way is a right to make
a way over a piece of land,
31) Consider the following statements
usually to and from another piece
with respect to GatiShakti Sanchar
of land.
Portal
• The portal has been developed by
1. The portal will provide single Madhya Pradesh State
window clearance for centralised Electronics Development
Right of Way (RoW) approvals. Corporation.
2. It is a collaborative institutional • This portal envisages
mechanism between all bringing transparency,
stakeholders including Central, accountability and
State and Local governments. responsiveness to all
stakeholders.

Www.iasbio.com | Www.exammap.com | WhatsApp for UPSC NOTES +918987187161


• The portal give impetus to ease • The mission objective is
of doing business. to conserve water, minimize
• Telecom service providers and wastage and ensure equitable
infrastructure providers will be distribution water within and
able to submit their applications across the states.
via this portal. • National Water Mission is a part
• Using this portal authorities will of National Action Plan on
be able to process and give Climate Change.
timely permissions for the • It will seek to ensure that a
deployment of digital network considerable share of the water

om
infrastructure. needs of urban areas are met
• This would provide a robust through recycling of waste water.
mechanism to achieve our goal of • The mission supports

.c
“Broadband to All” as envisaged development of low temperature
in the National Digital ap desalination technologies to
Communication Policy-2018. provide a reliable source of water
for coastal cities in India.
m
32) Consider the following statements • The 5 identified goals of the
with respect to National Water Mission Mission are:
m

o Comprehensive water data


1. It aims to conserve water, base in public domain.
xa

minimize wastage and ensure o Assessment of impact of


equitable distribution water climate change on water
within and across the states. resource.
.e

2. National Water Mission is a part o Promotion of citizen and


of National Action Plan on
w

state action for water


Climate Change. conservation,
w

3. The mission envisages increasing augmentation and


the water use efficiency by 20%. preservation and focused
W

attention to vulnerable
How many of the statements given areas including over-
above are correct? exploited areas.
o Increasing water use
• a. Only one
efficiency by 20%.
• b. Only two
o Promotion of basin level
• c. All Three
integrated water resources
• d. None
management.
• NAPCC has laid down the
Answer : c
National Water Mission principles and has identified the
Recently National Water Mission Inked approach to be adopted to meet
Pact with Indian Plumbing Association the challenges of impact of
to Improve Water Use Efficiency climate change through 8

Www.iasbio.com | Www.exammap.com | WhatsApp for UPSC NOTES +918987187161


National Missions which • a. Only one
includes: • b. Only two
o National Solar Mission, • c. Only three
o National Mission for • d. All four
Enhanced Energy
Efficiency, Answer : b
o National Mission on Small Savings Schemes
Sustainable Habitat, Recently the government of India has
o National Water Mission, made several changes to small saving
o National Mission for schemes which includes NRI being

om
Sustaining the Himalayan eligible to be nominate as nominee
Eco-system,
• The small savings scheme
o National Mission for a

.c
Green India, includes:
o Public Provident Fund
o National Mission for
Sustainable Agriculture
and
ap (PPF),
o National Saving
Certificate (NSC),
m
o National Mission on
o Sukanya Samriddhi,
Strategic Knowledge for
o Senior Citizens Savings
m

Climate Change.
Scheme,
o Mahila Samman Savings
xa

33) Consider the following statements


with respect to Small Savings Schemes Certificate Scheme,
o Kisan Vikas Patra, and
.e

1. Non-Resident Indians (NRIs) are o Post Office Monthly


not allowed to invest in small Income Scheme.
w

savings scheme. • The small savings are regulated


under Government Savings Bank
w

2. National Saving Certificate


(NSC), Sovereign Gold Bond Act, 1873.
W

Scheme (SGB) and Public • The interest rates of small


Provident Fund (PPF) are savings are revised quarterly.
examples of Small Savings • The recent changes made by
Schemes. government includes:
3. The recently launched Mahila o Non-resident Indians
Samman Savings Certificate (NRI) shall be eligible to
scheme is one of the small be nominated as
savings schemes. nominee subject to the
4. The interest rates for all savings condition that payment to
scheme remains the same. such nominee/s shall be on
non-repatriation basis.
How many of the statements given o Change can be made on
above are correct? Account’s name or

Www.iasbio.com | Www.exammap.com | WhatsApp for UPSC NOTES +918987187161


surname after approval of programme in collaboration
legal affidavit document. with UNDP and Food and
o Single Account can be Agricultural Organisation
subsequently converted (FAO).
into a Joint Account or • The project was launched in 1975
vice versa on account of in Odisha.
the death of an account • The project was later extended to
holder. Uttaranchal, Rajasthan, West
Bengal, Tamil Nadu, Andhra
34) Consider the following statements Pradesh, Gujarat, Kerala,

om
with respect to Indian Crocodile Madhya Pradesh, Maharashtra,
Conservation Project Andaman, Assam, Bihar and
Nagaland.

.c
1. The project focused on • This project successfully
conservation breeding to increase increased the population of
the estuarine crocodiles in India.
2. The project was launched in 1975
ap

estuarine crocodiles.
The increased crocodile
in Odisha.
m
population resulted in human-
3. It was launched by the Indian crocodile conflict.
government in collaboration with
m

• Especially
UNDP and Food and Agricultural in Bhitarkanika wildlife
xa

Organisation (FAO). sanctuary the head count of


estuarine crocodiles is 1,793 and
How many of the statements given it is home to nearly 70% of
.e

above are correct?


India's saltwater crocodile
w

• a. Only one population.


• b. Only two
w

35) Consider the following statements


• c. All Three with respect to 2023 IMO Greenhouse
W

• d. None Gas Strategy


Answer : c 1. The strategy aims to reduce
Indian Crocodile Conservation Green House Gas (GHG)
Project emissions from international
In recent times there is growing human-
shipping by at least 20% by
crocodile conflicts because of increased
2030.
crocodile due to Indian Crocodile
Conservation Project and increased 2. The strategy pushes use of
dependence on river biofuel and bio-blends in the
shipping industry.
• Indian Crocodile Conservation 3. The strategy was not backed by
Project is a captive India as it imposes more
breeding government run

Www.iasbio.com | Www.exammap.com | WhatsApp for UPSC NOTES +918987187161


restriction on developing • Strategy also says that the carbon
countries. intensity of international
4. Maritime shipping is responsible shipping should decline to reduce
for 3% of global anthropogenic CO2 emissions on average by at
GHG emissions. least 40% by 2030, compared to
2008 baseline levels.
How many of the statements given • According to
above are correct? the Intergovernmental Panel on
Climate Change 2022 report,
• a. Only one maritime shipping is responsible

om
• b. Only two for 3 % of global anthropogenic
• c. Only three GHG emissions and these
• d. All four emissions are continuing to grow

.c
rapidly.
Answer : c
2023 IMO Greenhouse Gas Strategy
Recently United Nations International
ap
36) Consider the following statements
with respect to Majorana
Maritime Organization (IMO) adopted
m
2023 IMO Greenhouse Gas Strategy 1. Fermions that are their own
which was backed by India antiparticles are called Majorana
m

fermions.
• 2023 IMO GHG Strategy aims
xa

2. All subatomic particles that make


to reduce the total annual GHG up matter are called fermions.
emissions from international 3. Majorana fermions help in
.e

shipping by at least 20% by building quantum computers.


2030, compared to 2008.
w

• Shipping industry to reduce the How many of the statements given


total annual GHG emissions from above are correct?
w

international shipping by at least


W

70%, striving for 80%, by 2040, • a. Only one


compared to 2008. • b. Only two
• The strategy pushes use • c. All Three
of biofuel and bio-blends in the • d. None
shipping industry.
• According to Strategy, near- Answer : c
zero emission technologies, fuels Majorana
and energy sources should
“represent at least 5% of the • Ettore Majorana is an
energy used by international Italian scientist who found that
shipping by 2030. the Dirac equation also allowed
• This Strategy was backed particles that satisfied certain
aggressively by India. conditions to be their own
antiparticles.

Www.iasbio.com | Www.exammap.com | WhatsApp for UPSC NOTES +918987187161


• Dirac equation described the Sangam Age
behaviour of subatomic particles Recently Tamil Nadu Archaeology
that moved at near the speed of Department have excavated few items
light. in Porpanaikottai in Tamil
• All subatomic particles that make Nadu’s Pudukottai district
up matter are called fermions.
• Fermions that are their own • The Sangam Age is a period that
antiparticles are called is dated from 300 BCE to 300
Majorana fermions. CE.
• Majorana zero modes can be • The recent excavated items in the

om
used to realise the powerful site includes:
topological quantum- o A gold stud, which might
computing that are much more be probably used as a nose

.c
resistant to errors than stud.
traditional computers. o A bone point, with pointed

• Algorithms running on a
quantum computer using
ap ends on both sides and
carvings, was probably
used for weaving.
m
Majorana zero modes will have
one more degree of freedom than o Red round-shaped
carnelian bead is an
m

those running on a computer that


doesn’t. evidence to domestic trade
xa

• Majorana zero modes are and


expected to exist at the ends of o A Sangam-age fort with
the nanowire, as a result of the three-course brick
.e

interactions between the structure.


materials’ electronic structures. • Over 150 antiques, including
w

potsherds, hopscotches, spouts,


w

37) Which of the following excavations pieces of glass bangles and


sites are linked to the Sangam Age? beads, a terracotta lamp, a coin, a
W

spindle whorl and rubbing stone,


1. Keeladi besides a couple of graffiti, have
2. Sisupalgarh been found at the site so far.
3. Sivagalai • Carnelian beads have been found
4. Adichanallur at Kodumanal and a few other
places in Tamil Nadu.
Choose the correct code. • The excavation sites of Sangam
includes:
• a. 1, 2 and 3 only o Keeladi
• b. 2, 3 and 4 only o Sivagalai
• c. 1, 3 and 4 only o Adichanallur
• d. 1, 2, 3 and 4 o Porpanaikottai
o Kodumanal.
Answer : c

Www.iasbio.com | Www.exammap.com | WhatsApp for UPSC NOTES +918987187161


38) ESKAPE, sometimes seen in the • Nosocomial infections are
news recently, refers to which of the infections acquired during the
following? process of receiving health care
that was not present during the
• a. Group of Bacteria resistant time of admission.
to antibiotics • Nosocomial infections are caused
• b. Mission of NASA to study by a variety of organisms,
dark energy including bacteria, fungi, viruses,
• c. UN Mission to prevent child parasites, and other agents.
trafficking

om
• d. AI tool to build humanoids 39) Consider the following statements
with respect to Nari Adalat
Answer : a

.c
ESKAPE 1. Nari Adalat are women only
A recent study has revealed that courts at the village level.
ESKAPE pathogens don’t include E.
coli bacteria
ap
2. It works under the aegis of the
National Commission for
Women.
m
• ESKAPE pathogens, a group of
bacteria that are highly resistant Which of the following statement(s)
m

to antibiotics. given is/are correct?


• The ESKAPE pathogens includes
xa

o Enterococcus faecium, • a. 1 only


o Staphylococcus aureus, • b. 2 only
.e

o Klebsiella pneumoniae, • c. Both 1 and 2


o Acinetobacter baumannii, • d. Neither 1 nor 2
w

o Pseudomonas aeruginosa
and Answer : a
w

o Enterobacter species. Nari Adalat


W

• ESKAPE are able to resist Nari adalat are about to be set up on


multiple drugs and exhibit pilot basis in 50 villages each in Assam
physical characteristics that E. and Jammu and Kashmir from August
coli does not, such as the ability
• Nari Adalat is a women only
to remove their cell wall and
evade certain drugs. courts at the village level.
• ESKAPE pose a massive global • Nari Adalat (women’s court)
health threat because they will address individual cases and
can quickly evolve traits that also raise awareness about social
allow them to evade immune schemes under the government.
• The Nari Adalat does not hold
systems and available treatments.
• ESKAPE are the leading cause any legal status.
• The scheme would be run by the
of nosocomial
infections throughout the world. Ministry of Women and Child

Www.iasbio.com | Www.exammap.com | WhatsApp for UPSC NOTES +918987187161


Development under the Sambal 2. Bihar and Rajasthan have the
sub-scheme of Mission Shakti. least score grade in the index.
• The implementation process will 3. It is released by the Ministry of
be done in collaboration with the Education.
Ministry of Panchayati Raj, the
Ministry of Rural Development How many of the statements given
and Common Service Centers above are incorrect?
operated by the Ministry of
Electronics and Information • a. Only one
technology. • b. Only two

om
• The Nari Adalat of each village • c. All Three
would have 7-9 members where • d. None
half of would be the elected

.c
members of the gram Answer : a
panchayat and the other half Performance Grading Index (PGI)
women with social status like
teachers, doctors and social
ap
2.0

• The PGI 2.0 structure comprises


m
workers, who would
of 1000 points across 73
be nominated by the villagers.
indicators grouped into 2
m

• Its main functions include raising


categories which are Outcomes,
awareness about the legal
Governance Management (GM).
xa

rights and entitlements of women


• These categories of PGI 2.0 are
and resolving cases falling within
further divided into 6
its jurisdiction.
.e

domains namely:
• The services provided will
o Learning Outcomes,
w

include alternate dispute


o Access,
resolution and grievance
o Infrastructure & Facilities,
w

redressal, counselling, evidence-


o Equity,
based decision making, pressure
W

o Governance Process and


group tactics, negotiation,
o Teachers Education and
mediation and reconciliation
Training.
with mutual consent for
• Daksh is the highest achievable
accessible and affordable justice.
grade of the index with scoring
40) Consider the following statements more than 940 points out of total
with respect to Performance Grading of 1000 points.
• Akanshi-3 is the lowest
Index (PGI) 2.0
grade with score up to 460 out of
1. The index assess both the
performance of schools and
colleges across India.

Www.iasbio.com | Www.exammap.com | WhatsApp for UPSC NOTES +918987187161


1000 points. o PRABAND portal and
o Vidyanjali Portal data.

41) Consider the following statements


with respect to Western Disturbance

1. Western Disturbance is a storm


• Chandigarh has topped the that originate within tropics with
index. low pressure and create
• Chandigarh is followed disturbance across tropics.

om
by Punjab, Kerala, Puducherry 2. Unlike monsoon, Western
and Tamil Nadu. Disturbances have no fixed time
• Meghalaya has least score grade of arrival.
in the index.

.c
3. Western Disturbance is
• In the least category Meghalaya associated with rainfall, snowfall
is followed by Arunachal
Pradesh and Mizoram.
ap and fog in northern India.

How many of the statements given


m
above are correct?
m

• a. Only one
• b. Only two
xa

• c. All Three
• d. None
.e

Answer : b
w

Western Disturbance
The recent foods in northern parts of the
w

India (Chandigarh and its adjoining


areas) is due Western Disturbance in
W

addition to monsoon rains

• A Western Disturbance (WD) is


a storm originating from
• The PGI 2.0 is completely the Mediterranean
aligned with following: region, completely different
o Unified District from the monsoon.
Information System for • A Western Disturbance is a storm
Education Plus (UDISE that originate outside
+), tropics with low pressure and
o National Achievement create disturbance across and
Survey (NAS), outside tropics.
o PM POSHAN portal,

Www.iasbio.com | Www.exammap.com | WhatsApp for UPSC NOTES +918987187161


• Disturbance means an area of • d. Neither 1 nor 2
“disturbed” or reduced air
pressure. Answer : b
• A WD is associated with Prevention of Insults to National
rainfall, snowfall and fog in Honour Act, (1971)
northern India. Recently few men were jailed after they
• The moisture which WDs carry were detained for allegedly not rising
with them comes from the for the National Anthem in Srinagar
Mediterranean Sea and/or from
• Section 3 of the Act prescribes
the Atlantic Ocean.

om
• The WD is known for moisture jail up to three years and or a fine
usually being carried in the for intentionally preventing the
upper atmosphere, contrary to its singing of the National

.c
tropical counterparts where the Anthem or causing disturbance to
moisture is carried in the lower any assembly engaged in such


atmosphere.
WDs are frequent, and
ap

singing.
Indian National Flag in the act
includes any picture, painting,
m
their spells can be long or short.
• WDs have no fixed time of drawing or photograph, or other
visible representation of the
m

arrival unlike monsoon which are


expected in July and lasts till Indian National Flag.
• The disrespect to the Indian
xa

September.
National Flag includes:
42) Consider the following statements o Indignity offered to the
.e

with respect to Prevention of Insults to Indian National Flag.


National Honour Act, 1971 o Dipping the flag in salute
w

to any person or thing.


o Flying the flag at half-mast
w

1. Under the act, Indian National


Flag includes any woven wool, except on occasions on
W

cotton and silk flags and doesn’t which the flag is flown at
include any other art works. half-mast on public
2. Under the act, using the flag as a buildings.
portion of costume worn below o Using the flag as a drapery
the waist of any person in any form except in State
constitutes the disrespect to funerals or armed forces or
National Flag. other para-military forces
funerals.
Which of the above statements is/are o Using the flag as a portion
correct? of costume, uniform or
accessory of any
• a. 1 only description which is worn
• b. 2 only
• c. Both 1 and 2

Www.iasbio.com | Www.exammap.com | WhatsApp for UPSC NOTES +918987187161


below the waist of any • Article 226 empowers the High
person. Courts to issue writs for the
o Putting any kind of enforcement of the Fundamental
inscription upon the flag. Rights and ordinary legal right.
o Allowing the flag to touch • The writs include habeas corpus,
the ground or the floor or mandamus, prohibition,
trail in water intentionally. certiorari and quo-warranto.
o Using the flag as a • High court may refuse to exercise
covering for a building. its writ jurisdiction since Article
o Intentionally displaying 226 is discretionary.

om
the flag with the “saffron” • A High Court does not have the
down. power to issue directions to the
Union Government to invoke

.c
43) Consider the following statements Article 355.
with respect to Article 226 • High courts under Article 226
1. Article 226 empowers the High
ap can declare a law
unconstitutional and invalid on
Courts to issue writs for the
m
the ground of contravention of
enforcement of the Fundamental any of the Fundamental Rights.
Rights and ordinary legal right.
m

• Article 355 imposes a duty on


2. Under Article 266, High Courts the Centre to ensure that the
xa

are empowered to issue government of every state is


directions to the Union carried on in accordance with the
Government to invoke Article provisions of the Constitution.
.e

355. • Article 355 imposes two duties


3. High courts may refuse to
w

on the Centre:
exercise its writ jurisdiction since
w

Article 226 is discretionary. 1. To protect every state against


external aggression and internal
W

How many of the statements given disturbance.


above are correct? 2. To ensure that the government of
every state is carried on in
• a. Only one accordance with the provisions of
• b. Only two the Constitution.
• c. All Three
• d. None • The decision of whether or not to
invoke Article 355 is a political
Answer : b
one, and it rests with the Union
Article 226
government.
Madras High Court recently ruled that
A High Court, under Article 226 does 44) Consider the following statements
not have the power to issue a direction
with respect to Nuclear Poison
to the Centre to invoke Article 355

Www.iasbio.com | Www.exammap.com | WhatsApp for UPSC NOTES +918987187161


1. They are neutron absorbers that 45) Consider the following statements
controls the nuclear reactor’s with respect to Ozone layer and Water
power output and prevent vapour
criticality. Statement - I: The increase in water
2. Xenon-135 is a neutron poison vapour in the stratosphere can be
substance. mitigated by reducing greenhouse gas
emissions.
How many of the statements given Statement - II: Greenhouse gases trap
above are correct? heat in the atmosphere, which can lead
to an increase in water vapour in the

om
• a. 1 only stratosphere.
• b. 2 only Which one of the following is correct in
• c. Both 1 and 2 respect of the above statements?

.c
• d. Neither 1 nor 2
• a. Both Statement-I and
Answer : c
Nuclear Poison
ap Statement-II are correct and
Statement-II is the correct
explanation for Statement-I
m
• Neutron absorbers are sometimes • b. Both Statement-I and
called "poisons" because they can Statement-II are correct and
m

have a negative effect on the Statement-II is not the correct


reactivity of a nuclear reactor. explanation for Statement for
xa

• Neutron absorbers are called Statement-I


poison which controls nuclear • c. Statement-I is correct but
.e

reactor power output and prevent Statement-II is incorrect


criticality. • d. Statement-I is incorrect but
w

• U.S.-built nuclear Statement-II is correct


reactor, Sturgis is the World’s
w

first floating nuclear power Answer : a


W

plant. Ozone layer and Water vapour


• Uranium-238 makes up 99.27% Recent study found that increase of
of naturally occurring uranium. water vapour in the stratosphere is not
• Common type of nuclear reactor likely to delay the recovery of the ozone
requires thermal neutrons to layer
trigger nuclear fission.
• Xenon-135 is a neutron poison • Primary source of stratospheric
substance which is also water vapour is the oxidation of
responsible for Chernobyl methane.
disaster. • The rising temperature due to
• Breeder reactors could produce climate change and increasing
more fissile material than they methane concentration are both
consumed. expected to inject more water
vapour into the stratosphere.

Www.iasbio.com | Www.exammap.com | WhatsApp for UPSC NOTES +918987187161


• So if there is reduction in • In India, Arhar is known by
greenhouse gases there will different name such as Tur, Red
reduction in water vapour in Gram, and Pigeon Pea.
stratosphere. • Arhar is the main pulse crop
• The stratosphere lies 10-50 km of Kharif.
above the Earth’s surface and • It is source of protein which is
very dry region of atmosphere. grown in tropical and sub-
• Stratosphere is home to the ozone tropical regions.
layer that shields surface • Arhar is an inter crop with
life from harmful ultraviolet soyabean, cotton, moong and

om
radiation. urad.
• Although man-made increases in • Arhar is a largely rain-fed crop,
CO2 are warming the surface and with hardly 8% of its total area

.c
lowermost atmosphere, CO2 coming under irrigation.
actually cools the stratosphere. • Unlike Moong and urad, Arhar is
• Global warming is projected
to heat the tropopause region.
ap not a short-duration crop as it
grows over 150-180 days and is
m
harvested only in December-
46) Consider the following statements January.
m

with respect to Arhar • The major Arhar producing


states are:
xa

1. Arhar is a largely rain-fed Kharif o Maharashtra,


crop. o Uttar Pradesh,
2. Arhar is not a short-duration o Gujarat,
.e

crop. o Madhya Pradesh,


3. Tamil Nadu and Kerala are the
w

o Karnataka and
leading cultivators of Arhar. o Andhra Pradesh.
w

• Famous varieties of
How many of the statements given Arha includes
W

above are correct? o Richa 2000


o Upas-120
• a. Only one
o ICPL-87
• b. Only two
• c. All Three 47) Consider the following statements
• d. None with respect to Productivity Growth in
India’s Manufacturing Sector
Answer : b
Arhar 1. Productivity measures output per
With improper monsoon Arhar could unit of input, such as labour,
pose huge problems to framers whoi capital or any other resource.
have sown since it is largely rain-fed
crop

Www.iasbio.com | Www.exammap.com | WhatsApp for UPSC NOTES +918987187161


2. Employment in India’s Southern and Eastern states have
manufacturing sector has been on the lowest.
the rise since 2016. • Paying better salaries and
3. Western and Central Indian states providing more benefits
have higher average productivity increases manufacturing
in manufacturing relative to productivity.
Southern and Eastern states. • Employment in India’s
manufacturing sector
How many of the statements given has fallen from over 51 million
above are correct? in 2016 to less 36 million in 2023

om
even as the country’s working-
• a. Only one age population continued to
• b. Only two grow by millions every year.

.c
• c. All Three
• d. None 48) Consider the following statements

Answer : b
ap
with respect to Protection of Plant
Varieties and Farmers Rights Act, 2001
Productivity Growth in India’s
m
Manufacturing Sector 1. The Act recognises farmers as
According to a recent study the plant breeders and permits them
m

productivity growth of India’s to register their varieties.


manufacturing sector has worsened in
xa

2. The Act was introduced after


the past decade India joined the World Trade
Organisation.
.e

• Productivity measures output per


unit of input, such as labour, How many of the statements given
w

capital or any other resource. above are correct?


• The growth rate of productivity
w

fluctuated between 10 to 15 % in • a. 1 only


W

the 1990s and 2000s. • b. 2 only


• Productivity growth began to • c. Both 1 and 2
stagnate after 2015. • d. Neither 1 nor 2
• There is wide disparity in the
Productivity Growth across the Answer : c
states. Protection of Plant Varieties and
• Manufacturing Farmers Rights Act, 2001
productivity in Madhya Pradesh Recently Delhi High Court reject
is higher than Tamil Nadu. PepsiCo India’s patent appeal for its
• Western and Central Indian unique Lays’ potato in purview of
states tend to have the highest Protection of Plant Varieties and
average productivity in Farmers Rights Act, 2001
manufacturing, while the

Www.iasbio.com | Www.exammap.com | WhatsApp for UPSC NOTES +918987187161


• Protection of Plant Varieties and • India has ratified the Agreement
Farmers Rights Act, 2001 on Trade Related Aspects of the
(PPV&FR) provides an effective Intellectual Property Rights.
framework to conserve and
encourage the development of Protection of Plant Varieties and
various plant varieties. Farmers' Rights Authority
• PPV&FR Act established
an effective system to safeguard • The authority was established for
and recognise the rights of the following purposes:
breeders, researchers and farmers o To establish an effective

om
to promote agricultural system for protection of
development in the country. plant varieties.
• It also facilitates the Indian seed o To protect the rights of

.c
industry to ensure the availability farmers and plant
of high-quality seeds and planting breeders.


materials to farmers.
According to Section 34 of the
ap o To encourage the
development of new
varieties of plants.
m
PPV&FR Act, the protection
granted to a breeder may be o To accelerate agricultural
development.
m

revoked by the authority on the


following grounds:
49) Consider the following statements
xa

o If the grant of the


with respect to Director of Enforcement
registration certificate is
Directorate
against public interest.
.e

o If the grant of a
1. Director of Enforcement
w

registration certificate is Directorate have fixed tenures of


based on incorrect 2 years with a maximum of 2
w

information furnished by annual extensions.


the applicant.
W

2. A Committee comprising Prime


o When the breeder does not
Minister, Opposition Leader and
provide the registrar with the Chief Justice of India
the required documents. recommends for service
o In case of failure of the
extension.
breeder to provide the
required seeds for Which of the above statements is/are
compulsory licence. correct?
o In case of failure to
comply with the acts, • a. 1 only
rules, regulations and • b. 2 only
directions issued by the • c. Both 1 and 2
Authority. • d. Neither 1 nor 2

Www.iasbio.com | Www.exammap.com | WhatsApp for UPSC NOTES +918987187161


Answer : d How many of the statements given
Director of Enforcement Directorate above are correct?
Recently Supreme Court invalidated the
third extension of service to Sanjay • a. Only one
Kumar Mishra, the Director of the • b. Only two
Enforcement Directorate (ED) • c. All Three
• d. None
• Chief of CBI and the Director of
the Enforcement Directorate Answer : b
(ED) have fixed tenures of 2 Scheme for minor rape survivors

om
years with a maximum of 3 Ministry of Women and Child
annual extensions. Development recently announced
• Five-member panel composed of Scheme for minor rape survivors to
support minor rape survivor

.c
the Central Vigilance
Commissioner and Vigilance
Commissioners has
to recommend for service
ap• The scheme aims to provide
integrated support and
extensions for Director of the assistance to girl child victims
m
Enforcement Directorate (ED). under one roof, facilitate
• According to High immediate, emergency and non-
m

Court, extension of tenure to emergency access to a range of


services, including
xa

officers should be done only


in rare and exceptional o Access to education,
cases and that such extensions o Police assistance, and
.e

should be for a short period. health care,


o Maternity, neo-natal and
w

50) Consider the following statements infant care and


with respect to Scheme for minor rape o Psychological and legal
w

survivors support.
W

• The scheme would operate under


1. The scheme seeks to assist minor the aegis of the Nirbhaya Fund.
girls who have been abandoned • The scheme provides insurance
by their families due to forced cover for the minor girl
pregnancies resulting from rape. victim and her new-born under
2. The funding is allocated from the one roof to enable access to
Sexual Assault Victim Assistance justice and rehabilitation.
Fund. • Any girl who is below 18 years
3. It is not mandatory for victims to of age and is a victim of rape as
possess First Information Report per the provisions of the POCSO
(FIR) to avail the benefits Act is eligible under the scheme.
provided by the scheme. • First Information Report (FIR)
is not mandatory for availing the
benefits under the scheme.

Www.iasbio.com | Www.exammap.com | WhatsApp for UPSC NOTES +918987187161


• The scheme would be available Answer : c
at the level of Child Care Atal Vayo Abhyuday Yojana
institutions (CCIs) up to the age (AVYAY)
of 18 and thereafter up to the age
of 23 years in aftercare facilities. • The Atal Vayo Abhyuday Yojana
• According to National Crime (AVYAY), launched by
Records Bureau there are 51,863 the Ministry of Social Justice
cases under the Protection of and Empowerment, is a
Children from Sexual Offences comprehensive initiative aimed
(POCSO) Act in 2021. at empowering senior citizens in

om
o Out of these, 64% of cases India.
were reported under • The main objective of the
Sections 3 and 5 Scheme is to improve the quality

.c
(penetrative sexual assault of life of the Senior
and aggravated penetrative Citizens by providing basic
sexual assault
respectively).
ap amenities like shelter, food,
medical care and entertainment
opportunities.
m
o 99% of the cases were
committed against girls. • National Action Plan for Senior
m

Citizen (NAPSrc) had been


51) Consider the following statements revamped, renamed as Atal
xa

with respect to Atal Vayo Abhyuday VayoAbhyuday Yojana


Yojana (AVYAY) (AVYAY) and subsumed in
April 2021.
.e

1. The scheme aims to assist and • Under the umbrella Scheme, Atal
empower the senior citizens. Vayo Abhyuday Yojana
w

2. The scheme is launched by the (AVYAY), an Integrated


w

Ministry of Social Justice and Programme for Senior Citizens


Empowerment. (IPSrC) provides financial
W

3. National Action Plan for Senior assistance to eligible


Citizen (NAPSrc) has been organizations for running and
revamped and renamed as Atal maintenance of Senior Citizen
VayoAbhyuday Yojana Homes/ Continuous Care Homes.
(AVYAY). • Toll-free number 14567 is
a national helpline
How many of the statements given number namely Elder line for
above are correct? Senior Citizens.
• Rashtriya Vayoshri Yojana
• a. Only one (RVY) is a component under
• b. Only two AVYAY.
• c. All Three
• d. None Rashtriya Vayoshri Yojana (RVY)

Www.iasbio.com | Www.exammap.com | WhatsApp for UPSC NOTES +918987187161


• RVY aims to provide eligible Navigation Satellite System (DGNSS)
senior citizens suffering from any ‘SAGAR SAMPARK’
of the age-related disability/
infirmity, with assisted living • SAGAR SAMPARK is
devices, which can restore near an indigenous Differential
normalcy in their bodily Global Navigation Satellite
functions. System (DGNSS) which aims to
• Financial criteria for boost digitalisation in maritime
beneficiaries are either the Senior sector.
Citizen belonging to ‘Below • DGNSS is a terrestrial based

om
Poverty Line’ (BPL) category enhancement that “corrects the
income below Rs. 15,000 per errors and inaccuracies in
month. the Global Navigation Satellite

.c
System (GNSS) and provides
52) Consider the following statements more accurate positioning
with respect to SAGAR SAMPARK
ap

information.
DGNSS at 6 locations will help
1. It is an indigenous Differential mariners in safe navigation and
m
Global Navigation Satellite will reduce the risk of collisions,
System (DGNSS).
m

groundings, and accidents in the


2. DGNSS is a terrestrial based port and harbour areas.
xa

enhancement that corrects the • It is an important Radio Aid to


errors and inaccuracies in the Navigation towards fulfilment of
Global Navigation Satellite international obligations of the
.e

System (GNSS). following:


3. The DGNSS system aims to o International Maritime
w

boost digitalisation in maritime Organisation (IMO),


w

sector o Safety of Life at Sea


(SOLAS) and
W

Which of the following statement(s) o International Association


is/are incorrect? of Marine Aids to
Navigation and Lighthouse
• a. Only one Authorities (IALA).
• b. Only two • The latest DGNSS system is now
• c. All Three able to transmit corrections of
• d. None of the above GPS and Global Navigation
Satellite System (GLONASS).
Answer : d
• The error correction accuracy has
SAGAR SAMPARK
been improved from 5 to 10
Recently Union Minister of Ports,
meters to less than 5 meters for
Shipping and Waterways, Shri
Sarbananda Sonowal inaugurated the 100 Nautical Miles from Indian
indigenous Differential Global coastlines.

Www.iasbio.com | Www.exammap.com | WhatsApp for UPSC NOTES +918987187161


53) Consider the following statements • The SEBI lists the following 7
with respect to Market Infrastructure institutions as systematically
Institutions (MII) important MMIs:
o Bombay Stock Exchange,
1. MMI is a financial entity that o The National Stock
provides infrastructure for Exchange,
running the stock markets. o the Multi Commodity
2. India International Exchange is Exchange of India,
an important MMI. o The Metropolitan Stock
3. The R.N. Malhotra committee is Exchange of India,

om
related to MMI reforms. o Central Depository
Services Ltd,
How many of the statements given o National Securities

.c
above are correct? Depository Ltd and
o Multi Commodity


a.
b.
Only one
Only two
ap Exchange Clearing
Corporation.
• c. All Three
m
• d. None 54) Consider the following statements
m

with respect to Atlantic menhaden


Answer : a
Market Infrastructure Institutions
xa

1. Atlantic menhaden are filter


(MII) feeders that feed on
phytoplankton and zooplankton.
.e

• MMI is a financial entity that


2. Menhaden are harvested for use
provides essential infrastructure
as fertilizers, animal feed, and
w

for running the daily operations


bait for fisheries.
of stock or capital markets.
w

• MMI includes Stock exchanges, How many of the statements given


depositories and clearing
W

above are correct?


houses.
• Bimal Jalan committee is • a. 1 only
related MMI reforms, • b. 2 only
whereas R.N. Malhotra • c. Both 1 and 2
committee is related to insurance • d. Neither 1 nor 2
reforms.
• Any failure of such an MII could Answer : c
lead to even bigger cataclysmic Atlantic menhaden & Chesapeake
collapses that may result in an Bay
overall economic downfall. In recent times there is reduction in
• India International Atlantic menhaden due to over fishing in
Exchange is not a systematically and around the Chesapeake Bay
important MMI.

Www.iasbio.com | Www.exammap.com | WhatsApp for UPSC NOTES +918987187161


• Atlantic Menhaden Found • a. 1 only
in eastern coastal and estuarine • b. 2 only
waters from Nova Scotia of • c. Both 1 and 2
Canada to northern Florida of • d. Neither 1 nor 2
USA.
• They are filter feeders, primarily Answer : a
consuming phytoplankton and International Ocean Discovery
zooplankton in the water column. Program (IODP)
• Menhaden are harvested for use
as fertilizers, animal feed, and • IODP is a multinational

om
bait for fisheries including blue research collaboration
crab and lobster. program that explores Earth's
• They are a major source history and dynamics.

.c
of omega-3 fatty acids. • The program uses ocean-going
• Overfishing of the Atlantic research platforms to recover
menhaden may be the root cause
for declining reproductive rates
ap data recorded in seafloor
sediments and rocks and to
monitor subsea floor
m
of ospreys.
• Chesapeake Bay belong to environments.
• IODP depends on facilities
m

eastern coast of USA.


• St. Lawrence River in the middle funded by three platform
xa

latitudes of North America is providers with financial


the largest estuary in the world. contributions from five additional
• The Chesapeake Bay is partner agencies.
.e

the largest estuary in the • India's Ministry of Earth


Science (MoES) is one of the
w

USA and the third largest in the


world. additional funding partners to the
w

program.
55) Consider the following statements • IODP depends on facilities
W

with respect to International Ocean funded by three platform


Discovery Program (IODP) providers:
o The U.S. National Science
1. IODP is a multinational research Foundation (NSF)
collaboration program that o Japan's Ministry of
explores Earth's history and Education, Culture, Sports,
dynamics. Science and Technology
2. India’s Department of Science (MEXT)
and Technology is one of the o The European Consortium
funding partner to the program. for Ocean Research
Drilling (ECORD).
Which of the above statements is/are • Additional Funding Partners
correct? includes:

Www.iasbio.com | Www.exammap.com | WhatsApp for UPSC NOTES +918987187161


o China's Ministry of • It is a biodiversity hotspot and
Science and Technology UNESCO World Natural
(MOST) Heritage Site.
o Australian-New Zealand • The plateau is made of igneous
IODP Consortium rock having thinner layer of soil.
(ANZIC) • It experiences a natural cycle of
o India's Ministry of Earth extreme conditions.
Science (MoES) • It becomes water logged in
monsoon rains, very dry during
56) Consider the following statements summer and also experience dry

om
with respect to Kaas Plateau winter conditions during winter.
• This region has a hot-semi arid
1. Kaas Plateau is located in climate year-round with an

.c
Maharashtra and falls in the average temperature between 19-
biosphere of the Western Ghats. 33 degrees Celsius.
2. It is a biodiversity hotspot and
UNESCO World Natural
ap
• Plateau seems to change the
colours after every 15- 20 days as
Heritage Site.
m
the cycle of flowering plants
3. The plateau is made of igneous progresses with the monsoon
rock having thinner layer of soil.
m

progress since June to October.


• The annual rainfall in this region
How many of the statements given
xa

is around 763 mm.


above are correct? • The major portion of the plateau
is Reserve Forest.
.e

• a. Only one
• Kaas plateau is listed under the
• b. Only two
w

Protection Working Circle.


• c. All Three
• Many of the endemic,
• d. None
w

endangered plants are found on


Plateau.
W

Answer : b
• Kaas Lake (built 100 years ago)
Kaas Plateau
A recent study of the sediments from a is a perennial source of Water
seasonal lake in the Kaas Plateau has supply for western part of Satara
indicated a major shift in the Indian city.
Summer Monsoons towards dry and
stressed conditions with low rainfall 57) Consider the following pairs
Exercises
• Kaas plateau is locally called as Participants
'Kas Pathar' or ‘Plateau of
Flowers'. 1. JIMEX
• The plateau is located in 23 –
the Maharashtra, Western Indian and Japan navy
Ghats.

Www.iasbio.com | Www.exammap.com | WhatsApp for UPSC NOTES +918987187161


2. SALVEX • IN and USN have been
23 – U participating in joint Salvage and
S and Indian navy EOD exercises since 2005.
3. SOUTHERN READINESS • The exercise saw participation
23 – Maldives and from both the navies which
Indian navy included the ships – INS
Nireekshak and USNS Salvor in
How many of the pairs given above are addition to Specialist Diving and
correct? EOD teams.

om
• a. Only one Op Southern Readiness 2023
• b. Only two
• c. All Three • The exercise Op Southern

.c
• d. None Readiness 2023 was conducted
by Combined Maritime Forces
Answer : b
JIMEX 23
ap•
(CMF).
It was aimed at
strengthening multilateral
m
• JIMEX 23 is the 7th edition of ties and enhancing cooperation
Japan India Maritime Exercise through CMF exercise.
m

(JIMEX). • Participating Navies


• JIMEX 23 was hosted by
xa

includes USA, Italy, UK,


the Indian Navy in the Bay of Seychelles Defence Forces &
Bengal. Marine Police and member
.e

• The exercise witnessed complex nations of EUNAVFOR


exercises, undertaken jointly by (European Union naval force).
w

the two navies. • INS SUNAYNA also participated


• Both sides engaged in advanced
w

in the exercise.
level exercises in all three • A combined yoga session was
W

domains of maritime warfare - organised on board with


surface, sub surface and air. participation of personnel from
Seychelles Defence Forces and
SALVEX 23 CMF along with the ship's crew.
• SALVEX - IN-USN (Indian Combined Maritime Forces (CMF)
Navy – US Navy) Salvage and
Explosive Ordnance Disposal • Combined Maritime Forces
Exercise. (CMF) is a multinational
• It is the 7th edition. initiative aimed at enhancing
• It was conducted in month of maritime security, countering
July at Kochi. piracy to ensure safety and
freedom of navigation in the
region.

Www.iasbio.com | Www.exammap.com | WhatsApp for UPSC NOTES +918987187161


• CMF’s main focus areas by Land Space, a private
are defeating terrorism, Chinese company.
preventing piracy, encouraging • Zhuque-2 is also known by
regional cooperation, and different names such as Y-
promoting a safe maritime 2 and ZQ-2.
environment. • It operates based on the same
• It comprises of 34 member principles as traditional launch
nations. vehicles but differs in terms of
• India is not a member nation of propellant choice and certain
CMF. design considerations.

om
• The 34 nations that comprise • Methane is a hydrocarbon
CMF are not bound by either a fuel that can be produced from
political or military mandate. various sources,

.c
including natural gas.
58) Consider the following statements • Methane offers several
with respect to Zhuque-2
ap advantages as a rocket fuel, such
as high energy density, good
1. It is the world’s first hydrogen-
m
combustion characteristics, and
based space rocket. availability.
2. It uses methane as the fuel and
m

liquid oxygen (LOX) as the 59) Consider the following statements


xa

oxidizer. with respect to Honey Bees


3. It was launched by the National
Aeronautics and Space 1. All honeybees are social insects
.e

Administration (NASA). and live together in nests or


hives.
w

How many of the statements given 2. Honey bees communicate


above are correct?
w

information to its fellow bees


through waggle dance.
W

• a. Only one 3. Queen bee regulate the hive's


• b. Only two activities by producing chemicals
• c. All Three that guide the behaviour of the
• d. None other bees.
Answer : a How many of the statements given
Zhuque-2 above are correct?
Recently, China launched the World’s
first methane-fuelled space • a. Only one
rocket, Zhuque-2 • b. Only two
• c. All Three
• Zhuque-2 is the world’s first • d. None
methane-liquid rocket launched
Answer : c

Www.iasbio.com | Www.exammap.com | WhatsApp for UPSC NOTES +918987187161


Honey Bees 1. Clarion-Clipperton Zone (CCZ)
A recent study finds that bees can use is a zone of abundant
their rudimentary brains to make rapid Polymetallic nodules.
good decisions 2. CCZ is a region in Pacific Ocean
that falls outside national
• Honey Bee is any of a group of jurisdictions.
insects in the family Apidae 3. CCZ is regulated by International
(order Hymenoptera) that in a Seabed Authority (ISA).
broad sense includes all bees that
make honey. How many of the statements given

om
• All honeybees are social above are correct?
insects and live together in nests
or hives. • a. Only one

.c
• Usually, there will be a single • b. Only two
queen in a hive. • c. All Three
• Queens regulate the hive's
activities by producing chemicals
ap • d. None

that guide the behavior of the Answer : c


m
other bees. Clarion-Clipperton Zone (CCZ)
• The honeybee is remarkable for
m

the dancing movements it • CCZ is a region in Pacific


Ocean with abundant
xa

performs in the hive to


communicate information to its polymetallic nodules.
fellow bees about the location, • According to the International
.e

distance, size, and quality of a Union for Conservation of Nature


particular food source in the (IUCN), the extraction of
w

surrounding area. mineral resources from the


ocean below 200 metres is called
w

• Honey Bees nest above ground.


• Honey Bees use deep sea mining.
W

their rudimentary brains to make • Polymetallic nodules contain a


better and faster decisions than variety of metals including
humans. manganese, iron, copper, nickel,
• A honey bee has a brain smaller cobalt, lead and zinc, and small
than a sesame seed. but significant concentrations of
• Investigations of the bees’ brains molybdenum, lithium, titanium,
will help develop smatter drones and niobium, among other.
that work autonomously. • Seamounts and underwater
mountains formed through
60) Consider the following statements volcanic activity are rich in
with respect to Clarion-Clipperton Zone minerals.
(CCZ) • Important metals like cobalt,
copper, nickel, and manganese,
used extensively in batteries, are

Www.iasbio.com | Www.exammap.com | WhatsApp for UPSC NOTES +918987187161


believed to be abundant in the 1. Namda is a traditional carpet
Clarion-Clipperton Zone (CCZ). made of sheep wool.
• CCZ falls outside national 2. The art form belongs to
jurisdictions and is regulated by Jammu and Kashmir.
the International Seabed 3. The declined art form is
Authority (ISA). revived through project
Namda under Pradhan Mantri
Vishwakarma Kaushal Samman
Scheme.

om
How many of the statements given
above are correct?

.c
• a. Only one
ap • b. Only two
International Seabed Authority (ISA) • c. All Three
• d. None
m
• ISA comprises of 167 Member
States and the European Union. Answer : b
m

• The ISA can grant exploration Namda Art


contracts only in international The Namda craft of Kashmir is being
xa

waters that is the area defined as successfully revived under a Skill


the seabed and subsoil beyond India’s Pilot Project as part of the
.e

the limits of national jurisdiction Pradhan Mantri Kaushal Vikas Yojana


by the United Nations (PMKVY)
w

Convention on the Law of the


Sea (UNCLOS). • Namda art is a rug made of
w

• The region beyond the outer sheep wool through felting


technique.
W

limits of the continental shelf


comprises around 50% of the • Namda is a type of
entire seabed. traditional Kashmiri felted
• India has been granted two carpet that is created using
exploration contracts from the sheep wool and has colourful
ISA for: hand embroidery.
o Polymetallic nodules and • Namada is said to have begun
o Polymetallic sulphides. in the 16th century during
Mughal Emperor Akbar.
• It has been practiced by
61) Consider the following statements the Pinjara and Mansuri
with respect to Namda Art communities and Sama
Muslims in Kachchh.

Www.iasbio.com | Www.exammap.com | WhatsApp for UPSC NOTES +918987187161


• Due to low availability of raw compliance with basic
material, lack of skilled requirements.
manpower and marketing
techniques, the export of Which of the following statement(s)
this craft has declined almost is/are correct?
100 %.
• a. 1 only
• The Namda project seeks to
• b. 2 only
revive and preserve rich
• c. Both 1 and 2
heritage associated with
• d. Neither 1 nor 2

om
Namda craft.
• The project is public-private Answer : d
partnership (PPP) model in the Anti-defection

.c
field of skill development, as it
is being implemented in • The 52nd Amendment Act of
collaboration with local
industry partners.
ap 1985 provided for the
disqualification of the
m
• Through this special project members of Parliament and
under Pradhan Mantri the state legislatures on the
m

Kaushal Vikas Yojana ground of defection from one


(PMKVY), the Ministry of Skill political party to another is
xa

Development and often referred to as the ‘anti-


Entrepreneurship (MSDE) has defection law’.
.e

designed short-term training • Any question regarding


curriculum to preserve this disqualification arising out of
w

endangered craft. defection is to be decided by


• This will also improve the the presiding officer of the
w

access of existing artisans of House.


W

Namda crafts cluster in • The decision by presiding


Kashmir and will improve their officer on anti-defection law
prospects of employability. is subject to judicial review.
• Representation of People Act,
62) Consider the following statements 1950 empowers Election
Commission of India to
1. In India, party constitution is
register political parties.
mandatory for registering as a
• The Supreme Court verdict
political party.
in Indian National Congress (I)
2. Election Commission of India is
v Institute of Social Welfare &
empowered to suspend
Ors (2002) make it
registration or deregister a
clear that ECI cannot
political party for non-
deregister a party for violating

Www.iasbio.com | Www.exammap.com | WhatsApp for UPSC NOTES +918987187161


the Constitution or for • Pakkasuran Malai is an eco-
breaching the undertaking sensitive area located in
given to it at the time of the Nilgiris of Tamil Nadu.
registration. • It is primarily composed of
• Party constitution is not rocks and grasslands is also
mandatory for registration of home to the rare Valeriana
political parties. leschenaultii, an endemic
species that is critically
63) Consider the following statements endangered.
with respect to Pakkasuran Malai

om
• Pakkasuran Malai is home to
range of rare species such as:
1. Pakkasuran Malai is an eco-
o Gunther’s burrowing
sensitive area located in the

.c
snake, Shaheen falcon,
Mahendragiri Hills.
Valeriana leschenaultia,
2. Droog Fort, a historic fort is ap Coonoor bush frog.
located in the peak of
• It is also home to large
Pakkasuran Malai.
m
mammals such as:
3. Pakkasuran Malai is an
o Nilgiri martens, mouse
important nesting site for the
m

deer, Indian gaurs,


Shaheen falcon and home to
barking deer, leopards,
the rare Valeriana
xa

sloth bears, Indian giant


leschenaultia.
flying squirrels, etc.
.e

How many of the statements given • Droog Fort, a historic fort on


above are correct? the peak of Pakkasuran Malai.
w

• Sacred groves are patches of


• a. Only one primeval forest that some rural
w

• b. Only two communities protect as


W

• c. All Three abodes of deities.


• d. None • Sacred groves help in the
protection of many rare,
Answer : b threatened and endemic
Pakkasuran Malai species of plants and animals.
Tamil Nadu Tourism Development
Corporation Limited (TTDC) plans to 64) Consider the following statements
open up the eco-sensitive area with respect to Chandrayaan-3
surrounding Pakkasuran Malai to
tourism may impact the biodiversity of 1. It is launched using the Polar
the area Satellite Launch Vehicle (PSLV).
2. The success of the mission will
make India the first country to

Www.iasbio.com | Www.exammap.com | WhatsApp for UPSC NOTES +918987187161


soft-land near the lunar South world to soft-land near the
Pole. lunar South Pole.
3. Like Chandrayaan-2, • Satish Dhawan Space Center
Chandrayaan-3 also uses the (SDSC) is the only spaceport in
Gravity Assist flyby technique. India to launch both satellites
and spacecraft.
How many of the statements given • Like Chandrayaan-
above are correct? 2, Chandrayaan-3 also uses
the Gravity Assist flyby
• a. Only one

om
technique.
• b. Only two
• The rocket is powered by a
• c. All Three
propellant, which is a mix of
• d. None

.c
fuel and oxidisers.
Answer : b • The payloads on the lander
Chandrayaan-3
ap and rover remain the same as
the Chandrayaan-2.
Recently Chandrayaan-3 was launched
m
from Satish Dhawan Space Center • The landing site of the latest
(SDSC) in Sriharikota mission is more or less the
m

same as the Chandrayaan-2:


• Chandrayaan-3 is India’s 3rd near the south pole of the
xa

unmanned mission which moon at 70 degrees latitude.


used indigenously developed • Chandrayaan-3 spacecraft is a
.e

technology. composite of 3 modules :


• The mission’s main objectives o A Propulsion module
w

are: (PM): Life – 3 to 6


o To demonstrate safe months
w

and soft landing on the o An indigenous Lander


W

lunar surface module (LM) - Vikram


o To demonstrate rover o A Rover – Pragyan
roving on the moon and • Chandrayaan-3 will not carry
o To conduct in-situ (on- an orbiter but it will use data
site) scientific from the Chandrayaan-2
experiments. orbiter.
• The mission is launched • India’s Chandrayaan missions
by Launch Vehicle Mark 3 take a month to reach the
(LVM3) is the heaviest rocket moon.
of ISRO. • The Chandrayaan-3, like
• The successful landing of the Chandrayaan-2, will take a
spacecraft will make India rather long interesting route to
the first country in the the Moon.

Www.iasbio.com | Www.exammap.com | WhatsApp for UPSC NOTES +918987187161


Answer : a
Salsola Oppositifolia Desfontania
Recently researchers from
Gandhinagar-based Gujarat Ecological
Education and Research (GEER)
Foundation has discovered a new
species of saltwort called Salsola
Oppositifolia Desfontania

• Salsola oppositifolia

om
Desfontania is species
of saltwort belonging to the
family of Amaranthaceae.

.c
• Saltworl is
ap a halophyte growing naturally
in the highly saline tracts of
the Indian desert.
m
• It is a perennial shrub that
grows in saline, arid to semi-
m

arid environments of
the Kutch district, Gujarat.
xa

65) Consider the following statements


with respect to Salsola Oppositifolia • It can grow one to two metres
Desfontania tall and have a smooth,
.e

cylindrical, woody base.


1. It is a perennial shrub that • It is a halophyte that does not
w

grows in saline, arid to semi- have any hairs.


w

arid environments of Tamil • The leaves of this plant grow


Nadu. opposite each other in the
W

2. It is the first salt water plant stem.


species discovered in India. • Salsola oppositifolia
3. It is used as a raw material for Desfontania is the sixth species
manufacturing soda ash. of Salsola genus to be
discovered in India.
How many of the statements given • Plants of the Salsola genus
above are correct? have salty juice stored in their
leaves.
• a. Only one
• Salsola oppositifolia is used as
• b. Only two
a raw material for
• c. All Three
manufacturing soda ash.
• d. None

Www.iasbio.com | Www.exammap.com | WhatsApp for UPSC NOTES +918987187161


• Salsola species are used in • Gibbons are the smallest and
manufacturing soaps. fastest of all the apes.
• Suaeda nudiflora and Suaeda • Like all apes, Gibbons
fruticosa are also halophytes are extremely intelligent,
and are used as fodder in with distinct personalities and
Kutch. strong family bonds.
• All 20 species of Gibbons are at
• Gujarat Ecological Education
a high risk of extinction.
and Research (GEER)
• The Lars, a group of species
Foundation is an autonomous
classified in the genus Hylobates,

om
body, set up in 1982.
are the smallest and have the
• It is set up by the Forests & densest body hair.
Environment Department,

.c
Government of Gujarat. Hoolock Gibbon
ap• Hoolock Gibbons is the India’s
only ape which are found
66) Consider the following statements
at North East India.
m
with respect to Hoolock Gibbons
• Hoolock gibbon is found from
Myanmar (Burma) west of the
m

1. Hoolock Gibbons are found in


India, Myanmar and Bangladesh. Salween River into Assam, India
xa

2. Hoolock gibbon is listed as and Bangladesh.


'Endangered' under the IUCN • They are at a high risk of
Red List. extinction.
.e

3. Gibbons is the smallest and


fastest of all apes. Global Gibbon Network (GGN)
w

• GGN was founded with a vision


w

How many of the statements given


above are correct? to safeguard and conserve
W

Gibbons which are key element


• a. Only one of Asia’s unique natural heritage.
• b. Only two • Launched in partnership with the
• c. All Three following organisations:
• d. None o Eco Foundation Global,
o The Hainan Institute of
Answer : c National Park,
Hoolock Gibbons o The IUCN Species
Recently Global Gibbon Network Survival Commission
(GGN) conducted an event in china to Section on Small Apes
address the decreasing number of (IUCN SSA) and
Gibbons due to deforestation o The Zoological Society of
Gibbons London

Www.iasbio.com | Www.exammap.com | WhatsApp for UPSC NOTES +918987187161


• The GGN focuses on • SDRF is used only for meeting
implementation and action the expenditure for providing
according to the 2030 Agenda for immediate relief to the victims.
Sustainable Development Goals, • SDRF is audited by
Global Biodiversity Framework the Comptroller and Auditor
and other global agendas’ targets. General of India (CAG).
• The Central Government
67) Consider the following statements contributes 75% of
with respect to State Disaster Response SDRF allocation for general
Fund (SDRF) category States/UTs and 90% for

om
special category States/UTs (NE
1. SDRF is a statutory body States, Sikkim, Uttarakhand,
constituted under the Disaster Himachal Pradesh, Jammu and

.c
Management Act, 2005. Kashmir).
2. It is audited by the Comptroller • A State Government may use up
and Auditor General of India
(CAG).
ap to 10 % of the funds available
under the SDRF for providing
3. Lightning is notified as a natural
m
immediate relief to the victims of
disaster under SDRF. natural disasters that they
m

consider to be ‘disasters’ within


How many of the statements given the local context in the State.
above are incorrect?
xa

• Natural Disasters notified under


SDRF: Cyclone, drought,
• a. Only one
earthquake, fire, flood, tsunami,
.e

• b. Only two
• c. All Three hailstorm, landslide, avalanche,
w

• d. None cloudburst, pest attack, frost and


cold waves.
w

• Lightning is not a notified


Answer : a
natural disaster under SDRF.
W

State Disaster Response Fund (SDRF)


The Union government recently • According to the National Crime
affirmed that it is not willing to declare Records Bureau (NCRB), in the
lightning a natural disaster as deaths year 2021, as many as 2,880
caused by it can be avoided through people died due to lightning.
education and awareness • There has been an increase in the
proportion of such deaths
• The State Disaster Response compared to the total accidental
Fund (SDRF), constituted under deaths caused by events related
Section 48 (1) (a) of the Disaster to nature.
Management Act, 2005, is the • Deaths due to lightning can be
primary fund available with State avoided through education and
Governments for responses to awareness.
notified disasters.

Www.iasbio.com | Www.exammap.com | WhatsApp for UPSC NOTES +918987187161


68) Consider the following statements • CPTTP has 11 countries which
with respect to Comprehensive and includes:
Progressive Agreement for Trans- o Australia, Brunei, Canada,
Pacific Partnership (CPTPP) Chile, Japan, Malaysia,
Mexico, New Zealand,
1. The CPTPP is a Free trade Peru, Singapore and
agreement (FTA) between the Vietnam.
Pacific Island Countries and the • India is not a member of
ASEAN countries. CPTTP.
2. All members of the CPTPP are • CPTPP requires countries to

om
also members of the Asia-Pacific eliminate or significantly reduce
Economic Cooperation (APEC). tariffs and make strong
3. India is not the member of commitments to opening services

.c
CPTPP. and investment market.
• It also has rules
How many of the statements given
above are correct?
ap addressing competition,
intellectual property rights and
m
protections for foreign
• a. Only one
companies.
• b. Only two
m

• CPTPP is seen as a bulwark


• c. All Three
against China’s dominance in the
• d. None
xa

region.
• The CPTPP recognises
Answer : a
the challenges facing Small and
.e

Comprehensive and Progressive


Agreement for Trans-Pacific Medium-sized enterprises
w

Partnership (CPTPP) (SMEs) in establishing export


Recently UK signed the markets, and includes outcomes
w

Comprehensive and Progressive to help make this task easier in


Agreement for Trans-Pacific the CPTPP region.
W

Partnership (CPTPP), biggest trade


deal since Brexit to provide tariffs 69) Consider the following statements
reduction for UK exports to Asia Pacific with respect to Kodava community
countries
1. Kodavas are indigenous people
• CPTTP will cut tariffs for UK of Kodagu region in Karnataka.
exports to Asia Pacific 2. Kodava Takke, the language of
countries and with UK Kodavas is classified as
membership, the trading bloc will endangered by the UNESCO.
have a combined GDP of 12 3. The Kailpodh festival is
trillion pounds and account associated with the Kodava
for 15% of global trade. community.

Www.iasbio.com | Www.exammap.com | WhatsApp for UPSC NOTES +918987187161


How many of the statements given • About 3,293 voters registered
above are correct? with the 111 recognized
monasteries in Sikkim State are
• a. Only one the only ones who can contest
• b. Only two and cast their votes for the
• c. All Three Sangha constituency seat.
• d. None
70) Consider the following statements
Answer : c with respect to TRISHNA Satellite
Kodava community

om
In recent times the Codava National 1. It is dedicated to climate
Council (CNC), a body fighting for monitoring and operational
autonomy for Kodava people is seeking applications.
for virtual constituency like Sangha

.c
2. It is jointly developed by the
virtual constituency due to their NASA and the ISRO.
decreasing numbers ap3. TRISHNA will operate in the
sun-synchronous orbit.
• Kodavas are indigenous people
m
of Kodagu region in the state How many of the statements given
of Karnataka. above are correct?
m

• Kodava language is a dialect


a mixture of Dravidian • a. Only one
xa

languages. • b. Only two


• The Kodavas have a long- • c. All Three
.e

standing martial tradition and • d. None


are known for their valour and
w

bravery. Answer : b
• Kodavas want a seat in both the TRISHNA
w

Karnataka Assembly and in


• TRISHNA - Thermal Infrared
W

Parliament similar to Sangha


virtual constituency. Imaging Satellite for High
Resolution Natural Resource
Sangha virtual constituency Assessment.
• TRISHNA is High-resolution
• Sangha Assembly constituency is satellite for Natural resource
one of the constituencies in the Assessment.
32-member Assembly of Sikkim. • It is jointly developed by French
• The Sangha constituency doesn’t Space Agency
exist on the map. (CNES) and Indian Space
• This seat is reserved for Research Organization (ISRO).
the Buddhist monastic • TRISHNA features two sensors,
community of Sikkim who are a Thermal InfraRed
Buddhist monks and nuns. (TIR) instrument and a Visible

Www.iasbio.com | Www.exammap.com | WhatsApp for UPSC NOTES +918987187161


and Near Infrared (VNIR)/Short • b. 2 only
Wave InfraRed (SWIR) sensor • c. Both 1 and 2
(VSWIR). • d. Neither 1 nor 2
• It is dedicated to climate
monitoring and operational Answer : d
applications. Operation Trinetra II
• TRISHNA will operate in a sun- Four militants killed in encounter in
synchronous orbit at an altitude J&K’s Poonch.
of 761 km.
• TRISHNA observations will • Operation Trinetra II launched in

om
enhance our understanding of Sindarah and Maidana of
the water cycle and improve Surankote tehsil, poonch district,
management of the planet’s Jammu and Kashmir.

.c
• It is a joint search operation by
precious water resources, to
better define the impacts of the Army and Police.
climate change, especially at
local levels.
ap• Random searches and surprise
checking of vehicles have been
intensified across Poonch.
m
• Launch is expected to be on Dec
2024.
72) Which of the following indices are
m

used in measuring the National


Multidimensional Poverty Index, 2023?
xa

1. School Attendance
.e

2. Cooking Fuel
3. Maternal Health
w

4. Child And adolescent Mortality


w

Choose the correct code.


W

71) Consider the following statements


with respect to Operation Trinetra II • a. 1 and 3 only
• b. 2 and 4 only
1. It is a joint operation of the • c. 2, 3 and 4 only
Indian Navy, Army and the Air • d. 1, 2, 3 and 4
Force.
2. It is a humanitarian operation that Answer : d
is carried out in the regions National Multidimensional Poverty
affected by flood. Index, 2023
Recently NITI Aayog released the
Which of the following statement(s) National Multidimensional Poverty
is/are correct? Index, 2023 to assess the status of
poverty level in India across states
• a. 1 only

Www.iasbio.com | Www.exammap.com | WhatsApp for UPSC NOTES +918987187161


• According to the index, India has 2. Child And adolescent
registered a significant decline of Mortality,
9.89% points in the number of 3. Maternal health,
multidimensional poor, from 4. Years of schooling,
24.85% in 2015-16 to 14.96% in 5. School attendance,
2019-2021. 6. Cooking fuel,
• 13.5 Crore people came out of 7. Sanitation,
multidimensional poverty 8. Drinking water,
between 2015-16 and 2019-21. 9. Electricity,
• The decline in poverty people is 10. Housing,

om
due to decrease in the number of 11. Assets, and
multidimensional poor in States 12. Bank accounts.
such as Bihar, Uttar Pradesh, • Rural areas witnessed the fastest

.c
Madhya Pradesh, Odisha, and decline in poverty over urban
Rajasthan. ap areas.
• Delhi, Kerala, Goa and Tamil
Nadu have the least number of 73) Consider the following statements
with respect to Crimean-Congo
m
people facing multidimensional
poverty along with the Union Haemorrhagic fever (CCHF)
m

Territories.
1. It is a zoonotic viral disease
xa

transmitted by ticks.
2. ELISA test used to detect HIV
can also be used to detect CCHF.
.e

3. There is no vaccine available for


either people or animals.
w

How many of the statements given


w

above are incorrect?


W

• a. Only one
• b. Only two
• The report has been prepared • c. All three
based on the latest National • d. None
Family Heath Survey of 2019-21
and is the second edition of the Answer : d
National Multidimensional Crimean-Congo haemorrhagic fever
Poverty Index. (CCHF)
• The index has 12 parameters of Due to climate changes the CCHF are
health, education, and standard of thriving in geographies that
living. traditionally had a climate hostile to
1. Nutrition, them

Www.iasbio.com | Www.exammap.com | WhatsApp for UPSC NOTES +918987187161


• CCHF is a viral haemorrhagic biocontrol agent to control
fever usually transmitted by mosquitos.
ticks. 2. They are found mostly in fresh
• The disease was first described in and brackish, and occasionally in
the Crimean Peninsula in 1944 marine waters.
and given the name Crimean 3. The species is native to the U.S.
haemorrhagic fever. and is categorised as invasive
• The virus is present in the tick species in India.
family of insects.
• The CCHF is endemic to Africa, How many of the statements given

om
the Balkan countries, Middle above are correct?
East, and parts of Asia.
• CCHF outbreaks constitute a • a. Only one

.c
threat to public health services as • b. Only two
the virus can lead to epidemics. • c. All three
• CCHF symptoms include fever,
muscle ache, dizziness, neck
ap • d. None

Answer : c
m
pain, backache, headache, sore
eyes and sensitivity to light. Gambusia Fish
In recent times states such as Andhra
m

• According to the US Centers for


Pradesh, Chandigarh and Uttar
Disease Control and Prevention
Pradesh have released Gambusia Fish
xa

(CDC) animals such as cattle,


in their waterbodies to control malaria,
goats, sheep and hares “serve as
dengue.
hosts for the virus.
.e

• Transmission to humans occurs • Gambusia fish is also known as


w

through contact with infected mosquitofish because they act as


ticks or animal blood. biocontrol agent to control
w

• CCHF can be transmitted from mosquitos.


one infected human to another by •
W

They are found mostly in fresh


contact with infectious blood or and brackish, and occasionally
body fluids”, such as sweat and marine waters.
saliva. • It is native to the waters of the
• There is no vaccine for the virus south eastern United States.
in either humans or animals, and • The fish has a high breeding
treatment generally consists of capacity.
managing symptoms. • Multiple countries, including
India have listed Gambusia as
74) Consider the following statements invasive species.
with respect to Gambusia Fish
• Gambusia also get aggressive in
1. Gambusia fish is also known as environments where they need to
mosquitofish because they act as compete with other species for
resources.

Www.iasbio.com | Www.exammap.com | WhatsApp for UPSC NOTES +918987187161


• International Union for Answer : b
Conservation of Nature declared Export Preparedness Index 2022
Gambusia as one of the 100 NITI Aayog releases the third edition of
worst invasive alien species in the ‘Export Preparedness Index (EPI)
the world. 2022’ for States/UTs.
• In India, mosquitofish affected
the ecosystem health of the lake • The index is aimed at assessing
after its introduction into the the readiness of the states in
Nainital Lake in the1990s to terms of their export potential
control malaria. and performance.

om
• Export Preparedness Index 2022
is released by NITI Aayog.
• Tamil Nadu has topped the index

.c
which is followed by
Maharashtra, Karnataka, Gujarat,
ap Andhra Pradesh, Odisha, West
Bengal and Kerala.
• Among hilly/Himalayan states,
m
Uttarakhand has ranked at the top
position and is followed by
m

Himachal Pradesh, Manipur,


xa

75) Consider the following statements Tripura, Sikkim, Nagaland,


with respect to Export Preparedness Meghalaya, Arunachal Pradesh
Index (EPI) 2022 and Mizoram.
.e

• Haryana topped the chart among


1. The index assesses the readiness the landlocked regions.
w

of the states in terms of their • In the category of union


w

export. territories/small states, Goa was


2. The report is released by the ranked first.
W

Ministry of Commerce and • Karnataka and Tamil Nadu have


Industry. the highest number of GI
3. Karnataka and Tamil Nadu have products being exported.
the highest number of GI • The ranking is based on 4 main
products being exported. pillars: Policy, Business
ecosystem, Export ecosystem and
How many of the statements given Export performance.
above are correct? • The index is also aimed at
promoting competition among all
• a. Only one states.
• b. Only two
• c. All three 76) Consider the following statements
• d. None with respect to Dark Stars:

Www.iasbio.com | Www.exammap.com | WhatsApp for UPSC NOTES +918987187161


1. They are stars that have existed • There are almost entirely
even before the formation composed of Hydrogen.
of conventional. • They would not have been as hot
2. Unlike ordinary stars, they can as the universe’s first generation
gain mass by accumulating gas of ordinary stars.
falling into them in space. • It was the nuclear fusion
3. Dark Stars are entirely made up occurring in the cores of those
of methane. stars that spawned elements
heavier than hydrogen and
How many of the above statement(s) helium.

om
is/are correct?
77) Consider the following statements
• a. Only one with respect to Endometriosis:

.c
• b. Only two
• c. All three 1. It is a disease caused by a
• d. None of the above ap sexually transmitted virus.
2. It is a disease in which tissue
Answer : b similar to the lining of the uterus
m
Dark Stars grows outside the uterus.
Webb telescope captures tantalizing 3. There is currently no known cure
m

evidence for mysterious ‘dark stars’. for endometriosis.


xa

• A dark star is a type of star that How many of the above statement(s)
may have existed early in the is/are correct?
.e

universe before conventional


stars were able to form and • a. Only one
w

thrive. • b. Only two


• One supermassive dark star is as • c. All three
w

bright as an entire galaxy. • d. None of the above


W

• Unlike ordinary stars, they would


be able to gain mass by Answer : b
accumulating gas falling into Endometriosis
them in space. A new study could help expand the
• They are made of atomic matter range of treatment options for
and powered by the little bit of endometriosis.
dark matter that’s inside them.
• Dark stars would be able to • Endometriosis is a disease in
achieve a mass at least a million which tissue similar to the lining
times greater than the sun and a of the uterus grows outside the
luminosity at least a billion times uterus.
greater, with a diameter roughly • It can cause severe pain in the
ten times the distance between pelvis and make it harder to get
Earth and the sun. pregnant.

Www.iasbio.com | Www.exammap.com | WhatsApp for UPSC NOTES +918987187161


• Endometriosis can start at a How many of the above statement(s)
person’s first menstrual period is/are correct?
and last until menopause.
• With endometriosis, tissue • a. Only one
similar to the lining of the uterus • b. Only two
grows outside the uterus. • c. All three
• This leads to inflammation and • d. None of the above
scar tissue forming in the pelvic
region and (rarely) elsewhere in Answer : c
the body. Agricultural Produce Market

om
• The cause of endometriosis is Committees (APMC)
unknown. NITI Aayog has proposed an alternative
• There is no known way to to the existing Agriculture Produce
Marketing Committee (APMC)

.c
prevent endometriosis.
marketing system in the agriculture
• There is no cure, but its
symptoms can be treated with
medicines or, in some cases,
ap
sector.

• They are marketing board


m
surgery. established by the state
governments.
m

• They are established in order to


eliminate the exploitation
xa

incidences of farmers by the


intermediaries.
.e

• Agricultural marketing is a State


subject.
w

• The APMC has Mandis in the


market area that regulate the
w

notified agricultural produce.


W

• APMC ensures worthy prices and


78) Consider the following statements timely payments to the farmers
with respect to the Agricultural Produce for their produce.
Market Committees (APMC): • The e-National Agriculture
Market (e-NAM) is a single-
1. It is a marketing board window service for any
established by respective state information and services related
governments. to APMC.
2. The e-NAM portal is a single-
window service for any 79) Consider the following statements
information and services related with respect to the SCALP Missile:
to APMC.
3. Agricultural marketing is a State 1. The missile is jointly developed
subject. by India, Britain and France.

Www.iasbio.com | Www.exammap.com | WhatsApp for UPSC NOTES +918987187161


2. The missile is built under the • In 1997, the French and the U.K.
trilateral Storm Shadow government placed contracts for
agreement. the development and production
3. The missiles can carry both the of SCALP and Storm Shadow
conventional explosives and missiles with MBDA units in
nuclear warheads. their respective countries.
• Besides France and Britain,
How many of the above statement(s) SCALP and Storm Shadow
is/are incorrect? missiles have been exported to
Italy, Greece and Saudi Arabia,

om
• a. Only one among others.
• b. Only two
• c. All three 80) Consider the following statements:

.c
• d. None of the above
1. A predicate offence is a crime
Answer : b
SCALP Missile
ap that is a component of a more
serious crime.
Russia is raging at France for sending 2. The investigation and
m
SCALP missiles to Ukraine. prosecution of the predicate
offence is done by the Central
m

• The SCALP and its variants are a Bureau of Investigation (CBI) or


series of air-launched, long-range
xa

the State Police.


and conventionally armed 3. Any statement made by an
missiles capable of striking accused to the police is
.e

targets deep in enemy territory. inadmissible, whereas a


• The missile has a range of more statement made to an ED
w

than 250 kilometres (155 miles) authority is admissible in courts.


and it can hit targets up to 560
w

km away, depending on How many of the above statement(s)


W

parameters such as its launch is/are incorrect?


altitude.
• Derived from the Apache project, • a. Only one
SCALP was developed in the late • b. Only two
1990s and is now built by • c. All three
European weapons manufacturer • d. None of the above
Matra BAe Dynamics (MBDA.)
• The initial version was christened Answer : d
Système de croisière Enforcement Directorate and
conventionnel autonome à longue Prevention of Money Laundering Act
portée (SCALP), which translates (PMLA), 2002
to long range autonomous
conventional cruise system.

Www.iasbio.com | Www.exammap.com | WhatsApp for UPSC NOTES +918987187161


• Predicate offenses in money of Investigation (CBI) or the
laundering refers to crime State Police.
component of a larger crime. • The police are required to
• In a financial context, the register a First Information
predicate offense would be any Report (FIR) for a cognisable
crime that generated monetary offence before conducting an
proceeds. investigation.
• The larger crime would be • ED authorities begin with search
money laundering or financing of procedures and undertake their
terrorism. investigation for the purpose of

om
• The PMLA vested a cadre of gathering materials and tracing
officers under the Directorate of the ‘proceeds of crime’ by
Enforcement (ED) with powers issuing summons.

.c
to prevent money laundering, • Any statement made by an
attach proceeds of crime, and ap accused to the police is
confiscate assets. inadmissible as evidence in court,
• It has been repeatedly held that whereas a statement made to an
m
the PMLA is a sui generis ED authority is admissible.
legislation, enacted to tackle • A copy of the FIR is accessible to
m

money laundering through white the accused, whereas the


collar crimes. Enforcement Case Information
xa

• According to Section 3 of the Report is seldom available.


PMLA, the act of projecting or
claiming proceeds of crime to be 81) Arrange the following countries
.e

untainted property constitutes the (most powerful to least powerful), based


on the recently released Henley Passport
w

offence of money laundering.


• Under the Schedule to the Index, 2023:
w

PMLA, a number of offences


under the Indian Penal Code and 1. India
W

other special statutes have been 2. Singapore


included, which serve as the basis 3. Japan
for the offence of money 4. United States
laundering. 5. China
• In other words, the existence of
Choose the correct code.
predicate offence is sine qua non
to charge someone of money • a. 2-3-4-5-1
laundering. • b. 5-4-3-2-1
• It is crucial to note that the • c. 1-2-3-4-5
investigation and prosecution of • d. 3-4-2-1-5
the predicate offence is done
typically by the Central Bureau Answer : a
Henley Passport Index 2023

Www.iasbio.com | Www.exammap.com | WhatsApp for UPSC NOTES +918987187161


London-based global citizenship and long slide down the index,
residence advisory firm Henley & dropping two places to the eighth
Partners has released the Henley spot.
Passport Index 2023.
Most open countries
• Created in 2006, the Henley
Passport Index ranks the • The Top 20 ‘most open’
passports of countries according countries are all small island
to the number of destinations nations or African states, except
their holders can visit without a for Cambodia.

om
prior visa. • There are 12 completely open
• It ranks passports based on their countries that offer visa-free or
power and mobility based on the visa-on-arrival entry to all 198

.c
analysis of exclusive data from passports in the world (not
the International Air Transport counting their own), namely:
Association (IATA).
o IATA maintains the
ap Burundi, Comoro Islands,
Djibouti, Guinea-Bissau,
world’s largest and most Maldives, Micronesia,
m
accurate database of travel Mozambique, Rwanda, Samoa,
information. Seychelles, Timor-Leste, and
m

• It assumes only the passport of Tuvalu.


xa

citizen of country and it is not for


diplomatic, emergency or India
temporary in nature.
.e

• India has climbed seven places


Rankings on Henley Passport Index 2023
w

to 80th rank from 87 last year


• Singapore is the most powerful though the number of countries
w

passport in the world, with its allowed visa-free access to Indian


W

citizens able to visit 192 travel passport holders remain


destinations out of 227 around unchanged.
the world visa-free. • However, India is among the
• Germany, Italy, and Spain worst countries for allowing visa-
occupy the second place. free entry.
Alongside Japan at the third
position are Austria, Finland, 82) Consider the following statements
France, Luxembourg, South with respect to Tiger Orchids:
Korea, and Sweden.
• Japan which was first last year, 1. The Tiger Orchids are native to
dropped to third place this year. India and China.
• The U.K. climbed two places to 2. They are the largest and tallest
occupy the fourth place, while orchid species in the world.
the U.S. continued its decade-

Www.iasbio.com | Www.exammap.com | WhatsApp for UPSC NOTES +918987187161


3. They are used in treating skin
rashes, and anemia.

How many of the above statement(s)


is/are correct?

• a. Only one
• b. Only two
• c. All three
• d. None of the above

om
Answer : b
Tiger Orchids

.c
Tiger orchids, largest orchid species,
bloom at Kerala’s Jawaharlal Nehru
botanic garden. ap
83) Consider the following statements
with respect to the Faster Adoption and
• The orchid variety is native to Manufacturing of Electric Vehicles
m
South East Asia. (FAME) scheme:
• They are native to Malaysia,
m

Indonesia, Thailand, Myanmar 1. The scheme is implemented to


and Laos. reduce the price of EVs by
xa

• Grammatophyllum speciosum, incentivising manufacturers


popular as the ‘Tiger Orchid,’ is through subsidies.
.e

the largest orchid species in the 2. The scheme works under the
world. aegis of the Department of Heavy
w

• The plant was listed by the Industry.


Guinness Book of World Records 3. To be eligible for the scheme, the
w

as the world’s tallest orchid. electric vehicle should be fitted


W

• They are so called because of with 50% of locally made parts.


their splendid flowers which
sport striking brown spots against How many of the above statement(s)
a yellow backdrop, bringing to is/are correct?
mind tigers.
• Flowering starts in June and lasts • a. Only one
till August. • b. Only two
• After 8-12 years of growth, • c. All three
Grammatophyllum speciosum • d. None of the above
produces flowers in alternate
years in its natural habitat. Answer : c
• Different parts of tiger orchids FAME Scheme
can be used as treatments for skin
rashes, fever, and also anemia.

Www.iasbio.com | Www.exammap.com | WhatsApp for UPSC NOTES +918987187161


Seven manufacturers were fined over 1. It consists of India’s first
not complying with local sourcing International Financial Services
norms under the FAME scheme. Centre (IFSC) and an exclusive
Domestic Tariff Area (DTA).
• The scheme is implemented by 2. It is located on the shores of the
the Department of Heavy Sabarmati River.
Industry, Ministry of Heavy 3. The Vandana Aggarwal
Industries Committee is associated with the
• The scheme is implemented to International Financial Services
reduce the price of EVs by Centre Authority (IFSCA).

om
incentivising manufacturers
through subsidies. How many of the above statement(s)
• To be eligible for the scheme, a is/are incorrect?

.c
manufacturing clause required
the vehicle to be fitted with 50 • a. Only one
per cent of locally made parts by
vehicle value.
ap •

b.
c.
Only two
All three
• d. None of the above
m
FAME Phase I
Answer : d
m

• Under the scheme, subsidy is Gujarat International Finance Tech


being given to 11 cities for (GIFT) City
xa

launching electric buses, taxis Foreign investors will need assurance


and three-wheelers. about a benign regulatory environment
.e

• The cities include Delhi, before the IFSC takes off.


Ahmedabad, Bangalore, Jaipur,
w

Mumbai, Lucknow, Hyderabad, • It is a business district near


Indore and Kolkata, plus two Ahmedabad in Gujarat.
w

cities, Jammu and Guwahati • The building has been


W

under special category. conceptualized as an iconic


structure, reflective of the
FAME Phase II growing prominence and stature
of GIFT-IFSC as a leading
• Government has approved Phase- International Financial Centre.
II of FAME Scheme with an • It consists of a multi-service
outlay of Rs. 10,000 Crore for a Special Economic Zone (SEZ),
period of 3 years commencing which houses India’s first
from 1st April 2019. International Financial Services
Centre (IFSC) and an exclusive
84) Consider the following statements Domestic Tariff Area (DTA).
regarding Gujarat International Finance • It is situated on the shores of the
Tech-City (GIFT City): Sabarmati River.

Www.iasbio.com | Www.exammap.com | WhatsApp for UPSC NOTES +918987187161


International Financial Services • d. Neither 1 nor 2
Centre Authority
Answer : c
• IFSCA is a statutory body Light Fidelity (Li-Fi)
established in 2020 under the The Institute of Electrical and
International Financial Services Electronics Engineers has formally
Centres Authority Act, 2019 to announced 802.11bb as a standard for
ensure inter-regulatory light-based wireless communications,
coordination within the financial or Li-Fi.
sector.

om
• Headquarters: At GIFT City, • It uses special LED (light-
Gandhinagar in Gujarat. emitting diodes) light bulbs as
• It is a unified authority for the routers and works on optical

.c
development and regulation of wireless communications (OWC)
financial products, financial technology.
services and financial institutions
in the International Financial
ap
• Li-Fi devices will deliver data
through visible, infrared, or
Services Centre (IFSC) in India. ultraviolet light, unlike Wi-Fi
m
• In 2021, it had constituted the routers, which use radio
frequencies to transmit data.
m

Vandana Aggarwal committee to


examine global best practices in • Li-Fi's speed is said to be 100
times faster than WiGig, the
xa

financing and leasing of ships.


• The committee also assigned the fastest Wi-Fi in the 60GHz
task of identifying opportunities frequency band, which can
.e

and devise a roadmap to enable achieve a maximum speed of


such activities from GIFT IFSC. 7GB per second.
w

• Li-Fi decreases eavesdropping


and jamming and facilitates
w

85) Consider the following statements


with respect to Light Fidelity (Li-Fi): centimeter-precision indoor
W

navigation.
1. It uses special LED light bulbs as • If a user wants to use Li-FI, they
routers to deliver data. need a Li-Fi-equipped LED bulb
2. Unlike Wi-Fi routers, which use in the room since light cannot
radio frequencies to transmit penetrate walls like radio waves.
data, Li-Fi devices use visible, • Currently, developers are
infrared or ultraviolet light. working to optimise mixed use of
Wi-Fi and Li-Fi for homes and
Which of the above statement(s) is/are offices use.
correct?
86) Consider the following statements
• a. 1 only with respect to Sandstorms:
• b. 2 only
• c. Both 1 and 2

Www.iasbio.com | Www.exammap.com | WhatsApp for UPSC NOTES +918987187161


1. It is a meteorological inflowing and ascending currents
phenomenon common in arid and of a whirlwind of a small radius.
semi-arid regions. • Haboob is a strong wind that
2. Sahara Desert is famous for its occurs primarily along the
large sand dunes known as southern edges of the Sahara in
singing dunes. Sudan.
3. Haboob is the term for a mini • It is associated with large
stand storm that forms on a beach sandstorms and dust storms and
and moves inland. may be accompanied by
thunderstorms.

om
How any of the above statement(s) • Dust devil is a small whirlwind
is/are correct? or air vortex over land, visible as
a column of dust and debris.

.c
• a. Only one
• b. Only two 87) Consider the following statements


c.
d.
All three
None of the above
ap
with respect to Dark Matter:

1. Dark matter is completely


m
Answer : a invisible and is entirely made of
Sandstorms baryons.
m

Hundreds sent to hospitals due to new 2. Baryons are combination of


wave of sandstorms in Iran.
xa

protons, electrons, and neutrons.


3. Dark energy exerts negative
• Sand is usually made of Silica. repulsive energy that acts in the
.e

Also called silicon dioxide opposite direction of gravity.


(SiO2), it is the compound that
w

makes up most of the earth’s How many of the above statement(s)


crust. is/are incorrect?
w

• Sand dunes are created when


W

wind deposits sand on top of each • a. Only one


other until a small mound starts • b. Only two
to form. • c. All three
• Sandstorm, also called dust • d. None of the above
storm, is a meteorological
phenomenon common in arid and Answer : a
semi-arid regions. Dark Matter
• They arise when a gust front or A massive galaxy with little to no dark
other strong wind blows loose matter is puzzling scientists.
sand and dirt from a dry surface.
• Sand spout is a pillar of sand, • Over 80% of all matter in the
similar in appearance to a universe is made up of dark
waterspout, raised by the strong matter.

Www.iasbio.com | Www.exammap.com | WhatsApp for UPSC NOTES +918987187161


• Dark matter is completely How many of the above statement(s)
invisible. It emits no light or is/are correct?
energy and thus cannot be
detected by conventional sensors • a. Only one
and detectors. • b. Only two
• Dark Matter could be composed • c. All three
of both baryons and non-baryonic • d. None of the above
matter, meaning, they could
consist different types of Answer : c
particles. Panel of Vice-Chairpersons of Rajya

om
• Generally dark matter is believed Sabha
to be made up of particles that do For the first time in the history of Rajya
not have a charge. Sabha, the Chairman nominates four
women parliamentarians to panel of

.c
• Visible matter, also called
vice-chairpersons.
baryonic matter, consists of ap
baryons, an overarching name for • The role of the Vice-chairperson
subatomic particles such as of Rajya Sabha is to represent all
m
protons, neutrons and electrons. states and protect the state's
• The visible universe is the result rights against the union.
m

of interactions of 4 fundamental • The Deputy chairperson is the


forces namely, strong nuclear head of the Panel of Vice-
xa

force, weak nuclear force, chairpersons.


electromagnetic force and • The Chairman of the panel has
gravitation.
.e

the power to nominate the


• Dark energy is a negative members of the panel in the
w

repulsive force, that acts opposite absence of the president from


to the direction of gravity. time to time.
w

• The chairman has the power and


88) Consider the following statements
W

authority to nominate a
with respect to the Panel of Vice-
Chairperson of Rajya Sabha: maximum of 6 vice-chairmen
from the house of the house.
1. The role of Vice-chairperson is to • The absence of the chairman
represent all states and protect does not prevent delegation and
the state's rights against the execution of this power; in case
union. of his absence, the same duty is
2. The Deputy Chairperson of Rajya delegated and executed by the
Sabha is the head of the Panel of deputy chairman of the house. .
vice-Chairperson. • The vice-chairperson of the panel
3. Member of both ruling and has the power to select people
opposition parties can be selected from the opposition parties for
to the panel. further nomination.

Www.iasbio.com | Www.exammap.com | WhatsApp for UPSC NOTES +918987187161


• The chairperson of this panel • The 440 N thrust LAM engines
belonging to Rajya Sabha is provide the main propulsive force
nominated by the members of during the ascending phase,
Rajya Sabha. while the RCS thrusters ensure
precise altitude correction.
89) Consider the following statements • With the successful completion
with respect to the Service Module of the hot test, the SMPS
Propulsion System (SMPS): demonstrated its performance in
the full configuration.
1. The service module of

om
Gaganyaan is a regulated bi- 90) Consider the following statements
propellant based propulsion with respect to Section 69 (A) of the IT
system. Act:

.c
2. The SMPS helps in performing
orbital injection of the spacecraft 1. It allows the government to issue
during the ascent phase. ap content-blocking orders to online
intermediaries.
Which of the above statement(s) is/are 2. Under the law, only the Centre
m
correct? Government has the powers to
issue content-blocking orders.
m

• a. 1 only
• b. 2 only Which of the above statement(s)
xa

• c. Both 1 and 2 is/are incorrect?


• d. Neither 1 nor 2
.e

• a. 1 only
Answer : c • b. 2 only
w

Service Module Propulsion System • c. Both 1 and 2


(SMPS) • d. Neither 1 nor 2
w

ISRO successfully tests Gaganyaan


Service Module Propulsion System
W

Answer : b
(SMPS). Section 69 (A) of the IT Act
The government has asked social media
• The service module of platforms to take down Manipur video.
Gaganyaan is a regulated bi-
propellant based propulsion • Section 69 of the IT Act allows
system. the government to issue content-
• It caters to the requirements of blocking orders to online
the Orbital Module, performing intermediaries.
orbit injection, circularisation, • These intermediaries include
on-orbit control, de-boost Internet Service Providers (ISPs),
manoeuvring, and SM-based telecom service providers, web
abort (if any) during the ascent hosting services, search engines,
phase. online marketplaces, etc.

Www.iasbio.com | Www.exammap.com | WhatsApp for UPSC NOTES +918987187161


• The Section requires the • d. None of the above
information or content being
blocked to be deemed a threat to Answer : c
India’s national security, Anticyclones & Heat Dome
sovereignty, or public order. The US National Oceanic and
• If the Centre of state government Atmospheric Administration (NOAA)
are satisfied that blocking the announced that the last month was the
content is necessary and Earth’s hottest June since the record
expedient it may order the keeping of global temperatures began
content to be blocked. 174 years ago.

om
• As per rules that govern these Anticyclone
blocking orders, any request
• An anticyclone, also known as a
made by the government is sent
high-pressure system, is

.c
to a review committee, which
essentially an area of high
then issues these directions. ap pressure in which the air goes
• Blocking orders issued under
downwards towards the Earth’s
Section 69 (A) of the IT Act are
surface.
m
typically confidential in nature.
• As the air sinks, its molecules are
compressed, which increases the
m

91) Consider the following statements


with respect to Heat Domes & pressure, making it warmer.
• This causes dry and hot weather.
xa

Anticyclones:
• The winds remain calm and
1. Anticyclone is essentially an area gentle during an anticyclone, and
.e

of high pressure in which the air there is almost no formation of


goes downwards towards the clouds because here the air sinks
w

Earth’s surface. rather than rises.


w

2. Anticyclone is the cyclone that


rotates clockwise in the Northern Heat Dome
W

Hemisphere and
• A heat dome, on the other hand,
counterclockwise in the
Southern. occurs when an area of high-
3. A heat dome, on the other hand, pressure stays over a region for
occurs when an area of high- days and weeks.
• It traps warm air, just like a lid
pressure stays over a region for
days and weeks. on a pot, for an extended period.
• The longer that air remains
How many of the above statement(s) trapped, the more the sun works
is/are correct? to heat the air, producing warmer
conditions with every passing
• a. Only one day.
• b. Only two
• c. All three

Www.iasbio.com | Www.exammap.com | WhatsApp for UPSC NOTES +918987187161


• Heat domes, if they last for a official languages in Articles 343
long period, may cause deadly to 351.
heat waves. • A Bench headed by Chief Justice
of India D.Y. Chandrachud said
92) Consider the following statements the subject of the petition
with respect to the Eighth Schedule of concerns a policy matter.
the Constitution: • The court referred to a 1997
reported decision of the Supreme
1. The Eighth Schedule to the Court in the Kanhaiya Lal Sethia
Constitution of India lists the case to note that to include or not

om
official languages of the Republic to include a particular language
of India. in the VIIIth Schedule is a policy
2. The Supreme Court cannot direct matter of the Union.

.c
the Centre to include a language • Generally speaking, the courts do
as an official language in the not, in exercise of their power of
Eighth Schedule of the
Constitution.
ap judicial review, interfere in
policy matters of the State.
3. Rajasthani is an official language
m
• Unless the policy so formulated
in the Eighth Schedule of the either violates the mandate of the
Constitution.
m

Constitution or any statutory


provision or is otherwise actuated
How many of the above statement(s)
xa

by mala fides. No such infirmity


is/are incorrect?
is present in the instant case.
• The Bench noted that the court
.e

• a. Only one
• b. Only two need not oblige a petitioner
w

• c. All three merely because the latter thinks


• d. None of the above the cause he or she represents is a
w

genuine one.
• The eighth schedule includes the
W

Answer : a
Eighth Schedule of the Constitution recognition of the following 22
The Supreme Court said it cannot direct languages:
the Centre to include Rajasthani as an o Assamese, Bengali,
official language in the Eighth Schedule Gujarati, Hindi, Kannada,
of the Constitution. Kashmiri, Konkani,
Malayalam, Manipuri,
• The Eighth Schedule to the Marathi, Nepali, Odia,
Constitution of India lists the Punjabi, Sanskrit, Sindhi,
official languages of the Republic Tamil, Telugu, Urdu,
of India. Bodo, Santhali, Maithili
• Part XVII of the Indian and Dogri are the 22
Constitution deals with the languages presently in the

Www.iasbio.com | Www.exammap.com | WhatsApp for UPSC NOTES +918987187161


eighth schedule to the material from a glancing collision
Constitution. with another object, among other
scenarios.
93) Consider the following statements • Dimorphos is located in the
with respect to Asteroid Dimorphos: asteroid belt between Mars and
Jupiter, the Main Asteroid Belt.
1. Dimorphos orbits the larger • The Kuiper Belt is a doughnut-
asteroid Didymos. shaped ring of icy objects around
2. Dimorphos is located in the the Sun, extending just beyond
Kuiper Belt of Asteroids. the orbit of Neptune.

om
3. This asteroid was used in testing
the NASA’s Voyager Mission. 94) Consider the following statements
with respect to the Prohibition of

.c
How many of the above statement(s) Electronic Cigarettes Act (PECA):
is/are correct?

• a. Only one
ap
1. E-Cigarette is a battery operated
device that emits a vaporized
• b. Only two solution to inhale.
m
• c. All three 2. PECA prohibits the production,
• d. None of the above manufacture, import, export,
m

transport, sale, distribution and


Answer : a
xa

advertisement of e-cigarettes in
Asteroid Dimorphos India.
Hubble telescope detects boulder- 3. PECA works under the aegis of
.e

leaving asteroid Dimorphos. the Ministry of Health and


Family welfare.
w

• The Hubble Telescope has


detected 37 boulders leaving the How many of the above statement(s)
w

asteroid Dimorphos, which is/are correct?


NASA crashed the half-ton
W

DART spacecraft into in • a. Only one


September last year. • b. Only two
• The DART mission was • c. All three
humanity’s first test of a • d. None of the above
planetary defence technique.
• It also marked the first time that Answer : c
our species successfully changed Prohibition of Electronic Cigarettes
the motion of a celestial object. Act (PECA)
• Dimorphos may have formed Online portal launched to report
from material shed into space by violation of ban on e-cigarettes.
the larger asteroid Didymos. E-Cigarettes
• The parent body may have spun
up too quickly or could have lost

Www.iasbio.com | Www.exammap.com | WhatsApp for UPSC NOTES +918987187161


• An electronic cigarette is a 1. Polioviruses are enteroviruses
battery-operated device that that are transmitted primarily by
emits a vaporized solution to the faecal-oral route.
inhale. 2. The Oral Polio Vaccine (OPV)
• These devices have various contains weakened polioviruses
names, including e-cigarettes, e- to induce an immune response in
hookahs, vaporizer cigarettes, a human body without causing
vapes, and vape pens. disease.
• They come in a range of shapes. 3. The Inactivated Polio Vaccine
Some look like USB drives and (IPV) contains inactivated

om
others look like pens, for polioviruses and is administered
example. by injection.

.c
PECA How many of the above statement(s)
is/are incorrect?
• The Act the production,
manufacture, import, export,
ap • a. Only one
transport, sale, distribution and • b. Only two
m
advertisement of e-cigarettes in • c. All three
India. • d. None of the above
m

• Any person who contravenes this


Answer : d
xa

provision will be punishable with


imprisonment of up to one year, Polio
or a fine of up to one-lakh rupees, The polio vaccination programme has
.e

or both. successfully reduced the prevalence of


• For any subsequent offence, the polio cases by 99.9% worldwide since it
w

person will be punishable with an was launched in 1988.


w

imprisonment of up to three
• Polioviruses are enteroviruses
years, along with a fine of up to
W

five lakh rupees. that are transmitted primarily by


• Under the Act, no person is the faecal-oral route.
allowed to use any place for the
OPV
storage of any stock of e-
cigarettes. • One is the oral polio vaccine
• If any person stores any stock of (OPV), a live attenuated vaccine,
e-cigarettes, he will be meaning it contains weakened
punishable with an imprisonment polioviruses (all three types-1, 2,
of up to six months, or a fine of and 3) to induce an immune
up to Rs 50,000 or both. response in a human body
without causing disease.
95) Consider the following statements:
• In addition to being inexpensive
and easy to administer, the big

Www.iasbio.com | Www.exammap.com | WhatsApp for UPSC NOTES +918987187161


advantage of the OPV is that it later joined Hindustan
triggers a robust immune Republican Association (HRA).
response in the gut lining (called
the mucosal immune response). How many of the statements given
• This response is like a strong above are correct?
guard at the main gate, the
virus’s point of entry, and is • a. Only one
excellent at preventing disease as • b. Only two
well as person-to-person • c. All Three
transmission of the virus. • d. None

om
• The mucosal immune response is
also in addition to the significant Answer : b
immune response in the blood Bal Gangadhar Tilak

.c
(the systemic immune response).
• Bal Gangadhar Tilak is a scholar,
• The OPV is also easier to
manufacture, as the seed virus
required for the vaccine is an
ap mathematician, philosopher, and
ardent nationalist.
• He founded (1914) and served as
m
attenuated (weakened form) virus
President of the Indian Home
Rule League.
m

IPV
• Mahatma Gandhi called Bal
Gangadhar Tilak, the Maker of
xa

• It contains inactivated
polioviruses (all three types) and Modern India.
is administered by injection. • In 1916, he concluded the
.e

• It induces a strong systemic Lucknow Pact with Mohammed


immune response, thus protecting Ali Jinnah, which provided for
w

against paralytic poliomyelitis, Hindu-Muslim unity in the


nationalist struggle.
w

without any risk of causing


VAPP or VDPV. • The trial and sentence earned him
W

the title Lokamanya (Beloved


96) Consider the following statements Leader of the People).
with respect to Freedom Fighters • Tilak aimed at swarajya
(independence) and attempted to
1. Bal Gangadhar Tilak founded persuade the Congress Party to
and served as President of the adopt his militant program.
Indian Home Rule League. • In the Mandalay jail, Tilak wrote
2. Mahatma Gandhi called Chandra the “Secret of the Bhagavadgit
Shekhar Azad, the Maker of which is also known as Bhagavad
Modern India. Gita or Gita Rahasya.
3. Chandrasekhar Tiwari
participated in non- cooperation Chandrasekhar Azad
movement led by Gandhi and

Www.iasbio.com | Www.exammap.com | WhatsApp for UPSC NOTES +918987187161


• The original name of Answer : c
Chandrasekhar Azad is Adjournment Motion
Chandrasekhar Tiwari. Recently Congress MPs in Parliament
• Chandrasekhar Azad is an Indian moved adjournment motion to discuss
revolutionary who organized and alleged sexual assault of two women in
led a band of militant youth Manipur
during India’s independence
movement. • It is introduced in the Parliament
• Chandrasekhar Tiwari to draw attention of the House to
participated in non- cooperation a definite matter of urgent public

om
movement led by Gandhi and importance.
later joined Hindustan • It needs the support of 50
Republican Association. members to be admitted.

.c
• Azad participated in several • The Parliament exercises control
violent crimes, notably the over the Executive through zero


Kakori train robbery (1925).
Azad played a key role in
ap hour, half-an-hour discussion,
short duration discussion,
adjournment motion, no-
m
reorganizing the HRA as the
Hindustan Socialist Republican confidence motion, censure
motion.
m

Association (HRSA) after most


of the HRA’s members had been • The discussion on an
xa

killed or imprisoned. adjournment motion should last


for not less than two hours and
97) Consider the following statements thirty minutes.
.e

with respect to Adjournment Motion • The right to move a motion for


an adjournment of the business of
w

1. The purpose of an adjournment the House is subject to the


w

motion is to allow a discussion following restrictions:


on a definite matter of urgent o It should raise a matter
W

public importance. which is definite, factual,


2. The Parliament exercises control urgent and of public
over the Executive through zero importance.
hour, half-an-hour discussion and o It should not cover more
adjournment motion. than one matter.
o It should be restricted to a
Which of the above statements is/are specific matter of recent
correct? occurrence and should not
be framed in general
• a. 1 only terms.
• b. 2 only o It should not raise a
• c. Both 1 and 2 question of privilege.
• d. Neither 1 nor 2

Www.iasbio.com | Www.exammap.com | WhatsApp for UPSC NOTES +918987187161


o It should not revive actual jurisdiction so that
discussion on a matter that investigation can begin.
has been discussed in the • Whereas FIRs have serial
same session. numbers assigned to them, Zero
o It should not deal with any FIRs are assigned the number ‘0’.
matter that is under • It was introduced on the
adjudication by court. recommendation of the Justice
o It should not raise any Verma Comittee formed at the
question that can be raised backdrop of the brutal Nirbhaya
on a distinct motion. gang rape in Delhi, 2012.

om
• The concept of Zero FIR is a
98) Consider the following statements beneficial tool for the women of
with respect to Zero FIR the country against crimes like

.c
sexual harassment and rape.
1. It is a first information report • The term first information report
registered by any police station
irrespective of the area where the
ap (FIR) is not defined in the Indian
Penal Code (IPC), Code of
offence is committed.
m
Criminal Procedure (CrPC), 1973
2. It was introduced based on the or in any other law.
recommendations of the Justice
m

• The term first information report


Verma Committee. (FIR) is defined in police
xa

regulations or rules, information


Which of the above statements is/are recorded under Section 154 of
correct? CrPC.
.e

• a. 1 only 99) Consider the following statements


w

• b. 2 only with respect to Ludwigia Peruviana


• c. Both 1 and 2
w

• d. Neither 1 nor 2 1. Ludwigia Peruviana is an aquatic


W

weed native to India.


Answer : c 2. It is a flagship plant species that
Zero FIR (First information report) grows fast along water bodies in
In the ongoing Manipur violence the
the Western Ghats region.
number of Zero FIRs have increased
significantly Which of the above statements is/are
correct?
• Zero FIR is registered
irrespective of territorial • a. 1 only
jurisdiction in cases of cognisable • b. 2 only
offences. • c. Both 1 and 2
• Zero FIRs are later transferred to • d. Neither 1 nor 2
the police station that has the
Answer : d

Www.iasbio.com | Www.exammap.com | WhatsApp for UPSC NOTES +918987187161


Ludwigia peruviana
Recently it has been found that
Ludwigia Peruviana are threatening
elephant habitats and foraging areas in
Valparai, a hill station of Tamil Nadu

• Ludwigia peruviana is an aquatic


weed native to some countries in
Central and South America,
including Peru.

om
• Ludwigia peruviana is a
perennial, sometimes deciduous,
wetland shrub that can grow to 3

.c
and 4 metres
• Ludwigia peruviana can clog
waterways, damage structures
and dominate native vegetation.
ap
• Peruvian has poor wildlife value.
m
However it does form small
floating islands that can provide
m

refuge for water birds. 100) Batagaika Crater, located in



xa

These swamps are known for ________, is seen in the news recently
excellent grass covers, sedges for which of the following reasons?
and water sources that are very
.e

good for herbivores like gaur and • a. Moon; Meteor impact


elephant in particular. • b. India; Land subsidence
w

• Ludwigia is among the 22 • c. Russia; Permafrost thawing


w

priority invasive plants in Tamil • d. Argentina; Ground water


Nadu, the State’s drive to remove extraction
W

exotic species from its forests is


now largely focused on Lantana Answer : c
camara, Senna spectabilis and Batagaika Crater
Acacia mearnsii (wattle).
• Batagaika Crater is the World's
Most of these swamps are located in biggest permafrost crater located
private estates. in Far East of Russia.
• Base of the depression, marked
by irregular surfaces and small
hummocks, which began to form
after the surrounding forest was
cleared in the 1960s.

Www.iasbio.com | Www.exammap.com | WhatsApp for UPSC NOTES +918987187161


• It developed in the 1970s, first as • a. Only one
a ravine. Then by thawing in the • b. Only two
heat of sunny days, it started to • c. All Three
expand. • d. None
• Scientists say Russia is warming
at least 2.5 times faster than the Answer : c
rest of the world. Soliga tribes
• The locals call it “gateway to the
underworld” while scientists call • Soligas are an indigenous tribe of
it as mega-slumps. Karnataka.

om
• The soil beneath the slump, • The Soligas are the first tribal
which is about 100 metres deep community living in the core area
in some areas, contains an of a Biligiri Ranganatha Temple

.c
"enormous quantity" of organic (BRT) Tiger Reserve in India to
carbon. get their forest rights recognised.
• It will release into the
atmosphere as the permafrost
ap• Traditionally they have been
dependent on the forests for their
livelihood.
m
thaws, further fuelling the
planet's warming. • The Soligas are also called the
children of bamboo.
m

Silver Cockscomb
xa

• Silver cockscomb is a nutritious


.e

leafy green vegetable that grows


well even on fallow land and in
w

drought-like conditions.
• The scientific name is Celosia
w

argentea.
W

101) Consider the following statements • It is a short-lived 50-60 cm-tall


with respect to Soliga Tribe plant that bears simple, spirally
arranged leaves around the stem
1. Soligas are an indigenous tribe of with pinkish or silky white
Karnataka. flowers.
2. They live in the core area of • For the Soligas, known for their
Biligiri Ranganatha Temple traditional knowledge of ecology,
Tiger Reserve. silver cockscomb is a nutritious
3. Soliga tribes consume Silver
cockscomb, a weed as nutritious
leafy vegetable.

How many of the statements given


above are correct?

Www.iasbio.com | Www.exammap.com | WhatsApp for UPSC NOTES +918987187161


leafy green vegetable. Achievements
and Data Handling System.
• It is a step towards leveraging
technology to enhance efficiency
and manage the implementation
of Samagra Shiksha.
• It is for transparency and
accuracy in the system with
respect to approvals, releases
financial position.

om
• Districts can submit the physical
and financial expenditure on the
PRABANDH portal.

.c
Samagra Shiksha Scheme
ap• It is a Centrally Sponsored
Integrated Scheme for School
m
Education.
• It envisages the school as a
m

102) Consider the following statements


with respect to PRABANDH Portal continuum from pre-school,
xa

primary, upper primary,


1. The portal leverages technology secondary to Senior Secondary
to enhance efficiency and levels.
.e

manage the implementation of • It subsumes the 3 erstwhile


Schemes of Sarva Shiksha
w

Samagra Shiksha Scheme.


2. The portal works under the aegis Abhiyan (SSA), Rashtriya
w

of the Ministry of Education. Madhyamik Shiksha Abhiyan


(RMSA) and Teacher Education
W

Which of the following statement(s) (TE).


is/are correct? • Goal – To improve effectiveness
of school, measured in terms of
• a. 1 only equal opportunities for schooling
• b. 2 only and equitable learning outcomes.
• c. Both 1 and 2 • Vision – To ensure inclusive and
• d. Neither 1 nor 2 equitable quality education from
pre-school to senior secondary
Answer : a stage in accordance with the
PRABANDH Portal Sustainable Development Goal
(SDG) for Education.
• PRABANDH
- PRoject Appraisal, Budgeting,

Www.iasbio.com | Www.exammap.com | WhatsApp for UPSC NOTES +918987187161


103) Which of the following • The National Mission on
schemes/measures seeks to reduce Sustainable Agriculture (NMSA):
methane emission? o It is implemented by
Ministry of Agriculture
1. The National Mission on and Farmers Welfare
Sustainable Agriculture (NMSA). o It seeks to provide resilient
2. National Innovations in Climate practices including
Resilient Agriculture (NICRA) methane reduction
project. practices in rice
3. National Livestock Mission. cultivation.

om
4. The Gobardhan scheme. • National Innovations in Climate
Resilient Agriculture (NICRA):
Choose the correct code. o It is a project under The

.c
Indian Council of
• a. 1 and 3 only
ap Agricultural Research
• b. 2 and 4 only
(ICAR).
• c. 2, 3 and 4 only
o It has developed several
• d. 1, 2, 3 and 4
m
technologies with
mitigation potential for
Answer : d
m

methane from rice


Methane
cultivation.
xa

• The Gobar (Galvanizing Organic


• Methane (CH4) is a hydrocarbon
that is a primary component of Bio-Agro Resources) scheme
supports biodegradable waste
.e

natural gas.
• Methane is also a greenhouse gas recovery and conversion of waste
w

(GHG). into resources and reduction of


• According to Global Methane methane emissions.
w

Tracker 2022 agriculture is the


104) Consider the following statements
W

largest source of methane


with respect to National Mission on
emission in India.
Monuments and Antiquities (NMMA)
Methane reduction measures taken
1. The NMMA was launched in
by India
2007 to document India’s
• Direct Seeded Rice system monuments and antiquities.
reduces methane emissions as it 2. The mission is being
does not involve raising implemented by the
nurseries, puddling and Archaeological Survey of India
transplanting. (ASI).
• National Livestock Mission aims 3. All Indian heritage items and
to reduce methane emissions cultural repositories should only
from livestock. be owned and maintained by the

Www.iasbio.com | Www.exammap.com | WhatsApp for UPSC NOTES +918987187161


Archaeological Survey of India • NMMA was launched in 2007 to
(ASI). document the India’s monuments
and antiquities.
How many of the statements given • It was mandated with the target
above are correct? to document 58 lakh of the
antiquities.
• a. Only one • It was initially set up for a period
• b. Only two of 5 years from 2007 to 2012.
• c. All Three • It was extended for a further 5
• d. None years till 2017 and later merged

om
with Archaeological Survey of
Answer : b India (ASI).
Due to slow pace of documentation • Archaeological Survey of India

.c
under the National Mission on
(ASI) is the nodal agency for
Monuments and Antiquities (NMMA)
ap NMMA to implement the various
the trafficking of Antiquities is on the
activities all over the country.
rise
Protection of Indian heritage Items
m
105) Consider the following statements
with respect to Cyber Surakshit Bharat
• Indian heritage items and cultural
m

Initiative
repositories which are scattered
all over the country come under
xa

1. It aims to train common people to


the purview of various agencies. safely use internet through
• Some are protected by ASI, some awareness campaigns.
.e

are under the state government 2. It is an initiative of Ministry of


and some are under trusts, local Electronics and Information
w

authorities and private Technology.


ownership.
w

• A big chunk of movable heritage Which of the above statements is/are


W

is unprotected and scattered all correct?


over the country without any
vigilance and protection. • a. 1 only
• Therefore, the documentation of • b. 2 only
all such antiquities in the form of • c. Both 1 and 2
digitized records along with • d. Neither 1 nor 2
maintenance and periodic
auditing of inventory is necessary Answer : b
to safeguard India’s tangible Cyber Surakshit Bharat
cultural heritage. Recently National e-Governance
Division (NeGD) under Cyber Surakshit
National Mission on Monuments and Bharat organised CISO Deep-Dive
Antiquities (NMMA) training programme at Central

Www.iasbio.com | Www.exammap.com | WhatsApp for UPSC NOTES +918987187161


Academy for Police Training to spread 1. It is launched by the Narcotics
awareness about cyber crimes Control Bureau.
2. It aims to tackle the issue of
• Cyber Surakshit Bharat aims to substance abuse with a vision to
spread awareness about cyber- make India drug free.
crime and build capacities of
Chief Information Security Which of the above statements is/are
Officers (CISOs) and frontline IT correct?
officials, across all government
departments. • a. 1 only

om
• It ensures adequate safety • b. 2 only
measures and defend the digital • c. Both 1 and 2
infrastructures by various • d. Neither 1 nor 2

.c
organizations and become future-
ready in tackling cyber-attacks. Answer : b
• It will include an awareness
program on the importance of
ap
Nasha Mukt Bharat Abhiyan

• Nasha Mukt Bharat Abhiyan was


cybersecurity; a series of
m
workshops on best practices. launched on 2020 to tackle the
• issue of substance abuse and a
m

This program also conducts


training sessions for CISOs and vision to make India drug free.
• It is presently being implemented
xa

technical officials to:


o The Central Government,
in 372 identified most vulnerable
o State Governments and
districts.
.e

• It was launched by the Ministry


Union Territories,
o Public Sector Banks and
of Social Justice &
w

Public Sector Units, Empowerment.


• Nasha Mukt Bharat Campaign is
w

o Defence Forces, Defence


PSUs and technical arms a three-pronged attack which
W

of the Air Force, Army includes:


and the Navy.
• CISO Deep-Dive training
programme seeks to spread
awareness, build capacity as well
as enable Government
departments on steps that need to
be taken to create a cyber- 107) Consider the following pairs
resilient eco system.
Geological eras key
106) Consider the following statements geologic events
with respect to Nasha Mukt Bharat
Abhiyan

Www.iasbio.com | Www.exammap.com | WhatsApp for UPSC NOTES +918987187161


1. Cenozoic • It lasted from 541 million
Era – Evolution of to 252 million years ago.
complex life • It was characterised by the
2. Precambrian Era – Rise evolution of complex life,
of modern animals including fish, plants,
3. Palaeozoic insects, and amphibians.
Era – Formation of 3. The Mesozoic Era
Earth • It lasted from 252 million
4. Mesozoic years until 66 million
Era – First years ago.

om
appearance of flowering plants • This was the age of
dinosaurs, together with
How many of the pairs given above are the first appearance of

.c
correct? birds and flowering plants
ap towards the denouement.
• a. Only one 4. The Cenozoic Era
• b. Only two • It began 66 million years
• c. Only three
m
ago and continues to this
• d. All four day.
m

• It is distinguished by the
Answer : b rise of modern animals.
Geological eras
xa

108) Consider the following statements


• Our planet is more than 4 billion with respect to no-confidence motion
.e

years old, so experts have divided


earth’s history into pieces of time 1. Any member of the Lok Sabha
w

called aeons, eras, periods and can move a no-confidence


epochs.
w

motion with a support of 50


• These divisions in Earth’s members of the house.
W

geological timescale demarcate 2. The passing of no-confidence


key geologic events and the motion leads to resignation of the
appearance (or disappearance) of ruling government.
notable forms of life. 3. The first no-confidence motion
• Broadly, there are 4 geological was moved against Prime
eras: Minister Indira Gandhi in 1973.
1. The Precambrian Era How many of the statements given
• It began 4.6 billion years above are correct?
ago with the formation of
our planet and the • a. Only one
emergence of the first life • b. Only two
forms. • c. All Three
2. The Palaeozoic Era • d. None

Www.iasbio.com | Www.exammap.com | WhatsApp for UPSC NOTES +918987187161


Answer : b • PM Indira Gandhi faced the most
No-Confidence Motion no-confidence motions (15),
Recently the opposition parties are in followed by Lal Bahadur Shastri
the verge of moving No-Confidence and PV Narasimha Rao (three
Motion against the ruling government each).
• The Atal Bihari Vajpayee
• A no-confidence motion is a government lost the no-
parliamentary process that allow confidence motion by a margin
the opposition to challenge the of one vote (269-270) in April
government's majority and ability 1999.

om
to govern. • The most recent no-confidence
• If the motion is passed, the motion was moved against the
government must resign. Narendra Modi government in

.c
• Any member of the Lok Sabha 2018.
can move a no-confidence
motion with a support of 50
members of the house.
ap
109) Consider the following statements
with respect to Silvopasture systems
• The passing of no-confidence
m
motion leads to resignation of the 1. Silvopasture is an ancient
ruling government. practice that harmoniously
m

• When the Lok Sabha passes a no- integrates trees, forage and
xa

confidence motion against the livestock on the same land.


council of ministers, all the 2. The tress grown in Silvopasture
ministers have to resign including Systems act as natural carbon
.e

those ministers who are from the sinks.


Rajya Sabha.
w

• The Rajya Sabha cannot remove Which of the following statement(s)


is/are correct?
w

the council of ministers by


passing a no-confidence motion.
W

• No-confidence motion need not • a. 1 only


state the reasons for its adoption. • b. 2 only
• The Speaker of the Lok Sabha • c. Both 1 and 2
will decide whether to admit the • d. Neither 1 nor 2
motion for discussion and debate.
• Since independence, 27 no- Answer : c
confidence motions have been Silvopasture Systems
moved in the Lok Sabha.
• Silvopasture is an ancient and
• The first no-confidence mot
proven practice that
moved against Prime
harmoniously integrates trees,
Minister Jawaharlal Nehru in
forage and livestock on the same
1963.
land.

Www.iasbio.com | Www.exammap.com | WhatsApp for UPSC NOTES +918987187161


• It also regulate local climatic 1. Indus Waters Treaty was signed
conditions, buffering against between India and Pakistan and
temperature and wind extremes, brokered by the World Bank.
providing a favourable living 2. The treaty gave the waters of the
environment for livestock. western rivers to India and the
• Trees on silvopasture lands act as eastern rivers to Pakistan.
natural carbon sinks, sequestering 3. The western rivers are Ravi,
significantly 5-10 times more Beas, and Sutlej and the eastern
carbon then pastures without rivers are Indus, Jhelum, and
trees, all while maintaining or Chenab.

om
enhancing productivity.
• The extensive root systems of How many of the above statements
trees within silvopasture plots is/are incorrect?

.c
contribute to nutrient cycling,
improved soil stability and • a. Only one
quality, while effectively
combating erosion.
ap •

b.
c.
Only two
All Three
• d. None
m
Answer : b
m

Indus River System


xa

• The Indus River rises in the south


west of Tibet Autonomous
.e

Region of China and flows


through the disputed Kashmir
w

region and then into Pakistan to


drain into the Arabian Sea.
w

• It is joined by numerous
W

tributaries, notably those of the


eastern Punjab Plain which are
the Jhelum, Chenab, Ravi, Beas,
and Sutlej rivers.
• The Indus River system has been
used for irrigation.

Indus Waters Treaty

110) Consider the following statements • Indus Waters Treaty was signed
with respect to Indus Waters Treaty on September 1960, between
India and Pakistan and brokered
by the World Bank.

Www.iasbio.com | Www.exammap.com | WhatsApp for UPSC NOTES +918987187161


• The treaty fixed and delimited • d. None
the rights and obligations of both
countries concerning the use of Answer : a
the waters of the Indus River PALNA Scheme
system.
• The treaty gave the waters of the • National Creche Scheme has
western rivers (the Indus, Jhelum, been revised and subsumed as
and Chenab) to Pakistan. Palna scheme.
• The eastern rivers (the Ravi, • It aims to provide for quality and
Beas, and Sutlej) were given to affordable day-care facilities for

om
India. the children (6 months to 6 years)
• Under the treaty India is also of working women.
obligated to let flow all the • The Palna component has been

.c
waters of the western rivers and included under Mission Shakti.
shall not permit any interference. • This scheme has dual role of
• Proprietary rights of eastern
rivers (Article II, Sutlej, Ravi
ap encouraging women’s
participation in work force and
improve nutrition and health
m
and Beas) vest with
India whereas proprietary rights status of children.
• The scheme is launched
m

of western rivers (Article III, the


Indus, Jhelum and Chenab) vest by Minister of Women and
xa

with Pakistan. Child Development.


• It enables more women to take up
111) Consider the following statements gainful employment amongst all
.e

with respect to PALNA Scheme socio-economic groups both in


the organized and unorganized
w

1. National Creche Scheme has sector, irrespective of their


w

been revised and subsumed as employment status.


Palna scheme to provide day-care
W

facilities for children of working 112) Consider the following statements


women. with respect to National Scheduled
2. It is a component of the Umbrella Caste Finance and Development
Scheme, Mission Vatsalaya. Corporation
3. The scheme was launched by the
Ministry of Social Justice and 1. It is a not-for-profit company
Empowerment. registered under section 8 of the
Companies Act, 2013.
How many of the statements given 2. It aims to provide Socio-
above are correct? Economic Empowerment to
Scheduled Caste persons having
• a. Only one annual family income up to Rs.3
• b. Only two lakh.
• c. All Three

Www.iasbio.com | Www.exammap.com | WhatsApp for UPSC NOTES +918987187161


3. The Corporation provides loan • It is engaged in providing
assistance for self-employment in financial assistance at
Agriculture and Allied Activities concessional interest rates under
sector. various credit schemes to
beneficiaries belonging to
How many of the statements given Scheduled Castes.
above are correct? • It provides funding through the
State/Union Territories
• a. Only one Channelizing Agencies and other
• b. Only two channel partners.

om
• c. All Three • It also extends various non-credit
• d. None schemes to support the target
groups.

.c
Answer : c
National Scheduled Caste Finance 113) Consider the following statements
and
(NSFDC)
Development Corporation ap
with respect ACROSS Scheme

1. It is an umbrella scheme that aids


m
• NSFDC is a not-for-profit in weather and climate
company registered under section forecasting.
m

8 of the Companies Act, 2013. 2. The scheme is being


• The corporation works under the
xa

implemented by the Ministry of


aegis of under the Ministry of Earth Sciences (MoES).
Social Justice and
.e

Empowerment. Which of the above statements is/are


• The objective of the corporation correct?
w

is to provide Socio-Economic
Empowerment to Scheduled • a. 1 only
w

Caste persons having annual • b. 2 only


W

family income up to Rs.3.00 • c. Both 1 and 2


lakh. • d. Neither 1 nor 2
• The Corporation provides loan
assistance for self-employment Answer : c
ventures in the sectors like: ACROSS SCHEME
o Agriculture and Allied
Activities, • ACROSS - Atmosphere
o Small Business/Artisan & & Climate Research-
Traditional Occupation, Modelling Observing Systems
o Service Sector (Including & Services.
Transport Sector) and • The schemes aids in weather and
o Education Loan for climate forecasting.
Technical & Professional • The Sub-scheme under the
Trades/Courses. umbrella scheme ACROSS are:

Www.iasbio.com | Www.exammap.com | WhatsApp for UPSC NOTES +918987187161


o Monsoon Convection, Which of the above statements is/are
Clouds, and Climate correct?
Change (MC4),
o High Performance • a. 1 only
Computing System • b. 2 only
(HPCS), • c. Both 1 and 2
o Monsoon Mission (MM- • d. Neither 1 nor 2
II),
o Atmospheric Observations Answer : c
Network, Global Education Monitoring Report

om
o Weather & Climate 2023
Services,
• The report is released by United
o Upgradation of Forecast
Nations Educational, Scientific

.c
System and
and Cultural Organization.
o Commissioning of
Polarimetric Doppler
Weather Radars (DWRs).
ap• The report is titled as
‘Technology in Education: A
Tool on Whose Terms’.
m
• The scheme is implemented by 4
• The lack of accessibility,
institutions under Ministry of
eligibility, reachability and
m

Earth Sciences (MoES) namely:


affordability has prevented
access to assistive technology in
xa

1. India Meteorological Department


(IMD) India.
• The report has endorsed banning
2. National Centre for Medium
.e

Range Weather Forecasting smartphones in schools in


(NCMRWF) situations where “technology
w

3. Indian Institute of Tropical integration does not improve


learning or if it worsens student
w

Meteorology (IITM)
4. Indian National Centre for Ocean well-being”.
W

Information Services (INCOIS) • It highlighted that “mere


proximity to a mobile device was
114) Consider the following statements found to distract students and to
with respect to Global Education have a negative impact on
Monitoring Report, 2023 learning in 14 countries.
• The digital education being
1. The report is released by United costly, could worsen the problem
Nations Educational, Scientific of unequal access in low-income
and Cultural Organization. countries.
2. The report has endorsed banning • Indiscriminate use of digital
smartphones in schools in technology in education would
situations. affect the child’s privacy.
• The report urged policymakers to
ensure child data protection laws

Www.iasbio.com | Www.exammap.com | WhatsApp for UPSC NOTES +918987187161


and accountability mechanisms • Parliamentary Privileges can
tailored to children. be enjoyed by the
• The report also mentions that parliamentary committees.
“banning mobile phones from • The Constitution has also
schools improves academic extended the parliamentary
performance, especially for low- privileges to those persons who
performing students”. are entitled to speak and take part
in the proceedings of a House of
Parliament or any of its
115) Consider the following statements committees.

om
with respect to Parliamentary Privileges o These include the attorney
general of India and Union
1. Parliamentary Privileges are not ministers.

.c
mentioned in the original • The parliamentary privileges do
constitution but provided in the not extend to the president who
rules of Lok Sabha.
2. It is extended to President and
ap is also an integral part of the
Parliament.
m
Attorney general of India when • The breach of Parliamentary
they speak or take part in the Privileges is punishable by the
m

proceedings of the House of respective Houses of the


Parliament. Parliament.
xa

3. Parliamentary Privileges cannot • Parliamentary privileges can be


be enjoyed by the parliamentary classified into 2 broad categories:
committees.
.e

1. Collective Privileges - Those that


How many of the statements given are enjoyed by each House of
w

above are correct? Parliament collectively.


w

2. Individual Privileges - Those that


• a. Only one are enjoyed by the members
W

• b. Only two individually.


• c. All Three
• d. None • The Article 105 of
the constitution mentions two
Answer : d parliamentary privileges that is
Parliamentary Privileges freedom of speech in Parliament
and right of publication of its
• Parliamentary privileges are proceedings.
special rights, immunities and
exemptions enjoyed by the two
Houses of Parliament, their
committees and their members. 116) Consider the following statements
with respect to World Cities Culture
Forum (WCCF)

Www.iasbio.com | Www.exammap.com | WhatsApp for UPSC NOTES +918987187161


1. It is a global network of cities study research on cities and
that share research and culture.
intelligence and explore the role • Bengaluru is the only Indian
of culture in future prosperity. city in the forum.
2. World Cities Report has been • The other WCCF cities are New
published by the WCCF York, London, Paris, Tokyo,
annually. Dubai, etc.
3. Hyderabad is the only Indian city • “World Cities Culture Forum
which joined the forum. Summit 2022” was held at
Helsinki, Capital of Finland.

om
How many of the statements given
above are correct? 117) Consider the following statements
Statement I: Snowball Earth Glaciation

.c
• a. Only one is thick sheets of ice that covered the
• b. Only two Earth for an extended period between


c.
d.
All Three
None
ap
700 to 500 million years ago.
Statement II: Snowball Earth
m
glaciation is the result of 2nd great
Answer : a oxygenation event.
World Cities Culture Forum
m

Which one of the following is correct in


Recently Bengaluru became the 1st respect of the above statements?
Indian city to join World Cities Culture
xa

Forum • a. Both Statement-I and


Statement-II are correct and
.e

• The World Cities Culture Forum Statement-II is the correct


(WCCF) was founded in 2012 by explanation for Statement-I
w

Justine Simons OBE, London’s • b. Both Statement-I and


Deputy Mayor for Culture & the
w

Statement-II are correct and


Creative Industries. Statement-II is not the correct
W

• The WCCF provides a way for explanation for Statement for


policy makers in 38 key cities to Statement-I
share research and intelligence, • c. Statement-I is correct but
and explore the vital role of Statement-II is incorrect
culture in their future prosperity. • d. Statement-I is incorrect but
• It provides a platform for city Statement-II is correct
leaders to share knowledge and
experience, and to develop Answer : c
innovative solutions to the Snowball Earth glaciation
challenges of urban life.
• The World Cities Culture • Snowball Earth glaciation is thick
Report is published every 3 sheets of ice that covered the
years and is the most Earth for an extended period
comprehensive dataset and case

Www.iasbio.com | Www.exammap.com | WhatsApp for UPSC NOTES +918987187161


between 700 to 500 million years • The CDMDF aids to help
ago. stabilize the Indian corporate
• It is one of the major glacial bond market in times of stress.
events in Earth’s history. • Corporate bonds are debt security
• Snowball Earth glaciation which is issued by a corporation
is followed by Second Great and issued to
Oxygenation Event. bondholders/investors.
• Second Great Oxygenation Event • CDMDF is an Alternative
resulted in increase in the amount Investment Fund (AIF) under
of oxygen in the Earth’s Securities and Exchange Board

om
atmosphere which eventually led of India (SEBI).
to the evolution of complex life • The mutual funds will have to
forms. contribute 25 basis points (bps)

.c
of the specified debt asset under
118) Consider the following statements management to the CDMDF.
with respect to Corporate Debt Market
Development Fund
ap
• The initial contribution of mutual
funds to CDMDF will be based
m
on Asset Under Management of
1. It is an Alternative Investment the specified Mutual Fund
Fund (AIF) under the aegis of
m

schemes as of last December-


Ministry of Corporate Affairs. end.
xa

2. The CDMDF will purchase • The CDMDF will purchase


investment grade corporate bonds investment grade corporate bonds
from fund houses in distress to from fund houses in distress to
.e

help them meet redemption help them meet redemption


requests.
w

requests.
• CDMDF shall purchase listed
Which of the following statement(s)
w

corporate debt securities from the


is/are correct?
specified debt-oriented Mutual
W

• a. 1 only Fund schemes.


• The fund aims to avert Franklin
• b. 2 only
• c. Both 1 and 2 Templeton-type debt fund crisis.
• Franklin Templeton debt fund
• d. Neither 1 nor 2
crisis occurred aftermath of the
Answer : b covid-19, since it couldn't sell
Corporate Debt Market Development most of its underlying lower-
Fund (CDMDF) rated securities.
SEBI has recently created framework
for CDMDF to protect the investors 119) Consider the following statements
investing in mutual funds with respect to Niger

Www.iasbio.com | Www.exammap.com | WhatsApp for UPSC NOTES +918987187161


1. Niger is a coastal country in West 1. Yamuna floodplains includes
Africa. farmlands, slums, colonies,
2. The equator passes through the flyovers and bridges.
Niger. 2. The Yamuna floodplain was
3. The Niger River originates in designated as a protected area
Niger. free from construction in the
Delhi Masterplan of 1962.
How many of the statements given 3. Yamuna’s floodplains are
above are correct? designated as ‘Zone O’ by Delhi
development authority.

om
• a. Only one
• b. Only two How many of the statements given
• c. All Three above are correct?

.c
• d. None
• a. Only one
Answer : d
Geography of Niger
ap •

b.
c.
Only two
All Three
Recently a group of military mutineers • d. None
m
in Niger announced that they had
overthrown the country's Answer : c
m

democratically elected president, Yamuna floodplains


Mohamed Bazoum The recent heavy rains in northern
xa

India have caused devastation in the


• Niger, officially called Republic floodplains of Delhi
.e

of Niger is a land locked country Floodplains


in western Africa.
w

• The country is located above the • A floodplain is an area adjacent


equator and tropic of cancer to a river or a stream that's
w

passes through it. subject to inundation whenever



W

The tropic of cancer passes the river swells up, such as


through 16 countries including during the rainy season.
India. • It is part of the riverine
• Niger is the third longest river in ecosystem and disturbance with it
Africa that originates in Guinea has the potential of adverse
highlands of Guinea, a costal ecological effects.
country in Africa. • Floodplains have a distinct
ecological role in the region they
are located in.
• Floodplains have dazzling arrays
120) Consider the following statements of biodiversity.
with respect to Yamuna floodplains • Humans have settled around
floodplains since ancient times as
the area has fertile soil that

Www.iasbio.com | Www.exammap.com | WhatsApp for UPSC NOTES +918987187161


supports agriculture, provides
riverine transportation and has
availability of drinking water for 121) Consider the following statements
housing and businesses. with respect to Poshan Bhi Padhai Bhi
• Floodplains are inseparable from (PBPB) Program
the river channel.
1. The program provides early
Yamuna floodplains childhood care and education
program at Anganwadi Centers.
• Yamuna is a lifeline to 5 States 2. The program is a component

om
and its floodplains are a charging under Mission Poshan 2.0.
point.
• The Yamuna floodplain was Which of the following statement(s)

.c
designated as a protected area is/are correct?
free from construction in the


Delhi Masterplan of 1962.
The Central Ground Water
ap •

a.
b.
1 only
2 only
Authority in 2000 also notified • c. Both 1 and 2
m
the floodplains as ‘protected’ for • d. Neither 1 nor 2
groundwater management.
m

• The floodplains are two km wide Answer : c


Poshan Bhi Padhai Bhi (PBPB)
xa

on each side of the river.


• The floodplain along Yamuna’s
• Poshan Bhi Padhai Bhi (PBPB) is
22 km run in Delhi, designated as
.e

‘Zone O’ by the Delhi an Early Childhood Care and


Development Authority. Education Program (ECCE) to
w

• Zone O supports a large variety help India develop the high-


quality pre-school network at
w

of nature-based livelihoods with


a low ecological footprint. AnganwadiCenters (AWCs).
W

• Under the programme, steps have


• Most residents migrated from
Uttar Pradesh, Bihar, Haryana, been taken to bolster early
Jharkhand, West Bengal and childhood care and education and
Rajasthan live on the Yamuna good nutrition practices through
flood plains. development.
• PBPB is not a separate scheme
• Muslim-dominated Batla House
area, a part of the floodplains that but is a programme component
falls under no-construction zones. under ECCE of Anganwadi
• The National Green Tribunal services under Mission Poshan
(NGT) in 2016 imposed a blanket 2.0.
• The implementation of PBPB
ban on agriculture-related
activities till “Yamuna is restored includes conducting Capacity
and made pollution free. Building trainings for Integrated

Www.iasbio.com | Www.exammap.com | WhatsApp for UPSC NOTES +918987187161


Child Development Services • c. All Three
(ICDS) functionaries. • d. None
• The objectives of the training
program include: Answer : a
o Promoting early DISHA Scheme
stimulation for the first
1000 days and ECCE for • DISHA
children 3-6 years of age. - Designing Innovative Solutions
o Developing early for Holistic Access to Justice.
childhood care and • It scheme aims to design and

om
education understanding of consolidate various initiatives to
Anganwadi Workers. provide citizen- centric delivery
o Emphasizing of legal services enunciated

.c
developmental domains under Articles 39A, 14 and 21 of
such as socio-emotional- the Constitution of India.
ethical, physical and
motor, cognitive, etc., and
ap
• The scheme is launched for a
period of five years 2021-2026 to
advance the cause of access to
m
the development of
Foundational Literacy and justice.
• The scheme was introduced
m

Numeracy (FLN).
o Reinforcing nutrition by Department of Justice.
xa

knowledge of Anganwadi • It aids to create awareness and


Workers. dissemination through use of
technology and developing
.e

122) Consider the following statements simplified Information,


with respect to Designing Innovative Education and Communication
w

Solutions for Holistic Access to Justice (IEC) material for the masses.
w

(DISHA) Scheme • The 3 subcomponents under the


scheme includes:
W

1. The scheme aims to provide free o Tele-Law: Reaching the


legal assistance and counsel to Unreached – An e-
the marginalized sections. interface platform that
2. The Tele-Law Service is a sub aims to empower people
component of the Scheme. through pre-litigation
3. The scheme was introduced by advice for rightfully
the Ministry of Social Justice and claiming their entitlements
Empowerment. and for timely redressal of
their difficulties.
How many of the statements given o Nyaya Bandhu (Pro
above are correct? Bono Legal Services) –
Aims to provide free legal
• a. Only one
• b. Only two

Www.iasbio.com | Www.exammap.com | WhatsApp for UPSC NOTES +918987187161


assistance and counsel to Microsites Project under
the marginalized sections. the Ayushman Bharat Digital
o Legal Literacy and Legal Mission (ABDM).
Awareness – Aims to • The primary goal of the
enable the vulnerable Microsites is to create small
sections of society to have ecosystems within specific
access to information geographic areas where
about their legal rights, complete ABDM adoption is
entitlements. achieved, ensuring the
▪ It also creates better digitization of the entire patient

om
legal awareness so journey.
as to make justice • Intended objectives of the
delivery system project

.c
citizen-centric. o Targeted adoption efforts

ap in private sector
123) Consider the following statements o Showcase end-to-end
with respect to 100 Microsites Project ABDM adoption
m
o On-Ground response and
1. Microsites are clusters of small feedback
and mid-sized health facilities to
m

• A Microsite is a focused region


provide digital health services to comprising of all small-medium
xa

patients. sized healthcare facilities such as


2. It was launched by the National clinics, hospitals, labs,
Health Authority (NHA) and pharmacies within that area,
.e

funded through the PM Cares which may be on-boarded onto


Fund.
w

the ABDM ecosystem.


3. The primary responsibility to run o Category A Microsite:
w

a microsite is with the Consists of at least 1000


States/Union Territories. Healthcare facilities
W

o Category B
How many of the statements given Microsite: Consists of 500
above are correct? to 1000 Healthcare
facilities
• a. Only one
• One State/UT can implement
• b. Only two
multiple Microsites.
• c. All Three
• The project will be implemented
• d. None
by State Mission Directors of
Answer : b Ayushman Bharat Digital
100 Microsites Project Mission.
• The financial resources and
• The National Health Authority overall guidance would be
(NHA) has introduced the 100

Www.iasbio.com | Www.exammap.com | WhatsApp for UPSC NOTES +918987187161


provided by National Health • Jammu and
Authority. Kashmir • Gujrat
• Primary responsibility to run a • Punjab • Sikkim
• Himachal • Tripura
Microsite is with the
Pradesh • Nagaland
States/UTs. • Haryana • Manipur
• Goa
124) Consider the following pairs
Tiger
Reserves Location • Udanti Sitanadi - Chattisgarh.
• Melghat - Maharashtra.

om
1. Udanti • Kawal - Telangana.
Sitanadi – Maharashtr • Bhadra - Karnataka.
a

.c
2. Sariska – Bih
ar
3. Kawal
ttisgarh
– Cha
ap

m
4. Bhadra Kar
nataka
m

How many of the pairs given above are


xa

correct?

• a. Only one
.e

• b. Only two
• c. Only three
w

• d. All four
w

125) It is a quasi-judicial constitutional


Answer : a
body and the recommendations of this
W

Tiger Reserve commissions are advisory and non-


binding in nature. The first Commission
• Tiger Reserve is a legally
was set up in 1951. The original
declared protected area dedicated jurisdiction of the Supreme Court does
to the conservation of Tiger. not extend to the matters referred to this
• Project Tiger was launched by commission.
the Government of India in the
The above statements describe which of
year 1973 to save the endangered
the following commissions?
species of tiger in the country.
• As on January, 2023 there are 53 • a. Election Commission
tiger reserve in India. • b. Union Public Service
Commission
States • c. Finance Commission
without a Tiger Reserve

Www.iasbio.com | Www.exammap.com | WhatsApp for UPSC NOTES +918987187161


• d. National Commission for • The 15th Finance Commission
Backward Classes was constituted on 27 November
2017.
Answer : c • Shri N.K. Singh is the chairman
Finance Commission of 15th Finance Commission.
• Criteria set by 15th Finance
• Article 280 provides for a Commission includes:
Finance Commission as a quasi- o Income distance - A state
judicial body. with lower per capita
• The Finance Commission income will have a higher

om
consists of a chairman and four share to maintain equity
other members to be appointed among states.
by the president. o Demographic

.c
• They hold office for such period performance - States with
as specified by the president in a lower fertility ratio will
his order and they are eligible for
reappointment.
ap be scored higher on this
criterion.
• The Constitution authorises the
m
o Forest and ecology - It
Parliament to determine the was arrived by calculating
qualifications of members of the
m

the share of the dense


commission and the manner in forest of each state in the
xa

which they should be selected. total dense forest of all the


• It is required to make states.
recommendations to the o Tax and fiscal efforts - It
.e

President on the following is measured as the ratio of


matters:
w

the average per capita own


o The distribution of the net tax revenue and the
w

proceeds of taxes to be average per capita state


shared between the Centre GDP during the three
W

and the states and the years between 2016-17


allocation between the and 2018-19.
states, the respective
shares of such proceeds. 126) Consider the following statements
o The principles which with respect to ULLAS (Understanding
should govern the grants- Lifelong Learning for All in Society)
in-aid to the states by the
Centre. 1. ULLAS is an interactive digital
o Any other matter referred gateway to engage in diverse
to it by the President in the learning resources through the
interests of sound finance. DIKSHA portal of NCERT.
2. The scheme aims to provide
15th Finance Commission digital literature to the students

Www.iasbio.com | Www.exammap.com | WhatsApp for UPSC NOTES +918987187161


belonging to the tribal knowledge-sharing in
community. communities across India.
3. The scheme is jointly • This initiative is poised to
implemented by the Ministry of revolutionise education and
Human resources and Minority literacy across the nation, by
Affairs. fostering a learning ecosystem
that reaches every individual,
How many of the statements given bridging the gaps in basic literacy
above are correct? and critical lifeskills.
• It imparts basic education, digital

om
• a. Only one and financial literacy and critical
• b. Only two life skills to citizens aged 15 and
• c. All Three above who lost on the

.c
• d. None opportunity to go to school.
• It is being implemented through
Answer : a
ULLAS (Understanding Lifelong
ap volunteerism.
Learning for All in Society)
m
127) Consider the following statements
Recently the Union Minister of with respect to All India Tiger
Education and Minister of Skill
m

Estimation Report, 2022


Development and
Entrepreneurship launched ULLAS
xa

1. The report provides the data on


mobile application the number of Tigers in each
state of India.
.e

• ULLAS is a user-friendly and 2. The report is released by the


interactive app that will serve as Ministry of Environment Forest
w

a digital gateway for learners to and Climate Change.


engage in diverse learning
w

3. The highest tiger mortality was


resources through the DIKSHA witnessed in the state of Odisha.
W

portal of NCERT.
• This app can be used for How many of the statements given
registration of learners and above are correct?
volunteers either through self-
registration or by surveyors. • a. Only one
• ULLAS will focus on promoting • b. Only two
functional literacy, vocational • c. All Three
skills, and many important life • d. None
skills like financial literacy, legal
literacy, digital literacy and Answer : a
empowerment of citizens. All India Tiger Estimation Report,
• It also fosters a culture of 2022
continuous learning and Recently All India Tiger Estimation
Report, 2022 was released by by junior

Www.iasbio.com | Www.exammap.com | WhatsApp for UPSC NOTES +918987187161


Union environment minister Ashwini completed five cycles of MEE of
Kumar Choubey along with Tiger Reserves in the country.
Uttarakhand chief minister Pushkar
Singh Dhami How many of the statements given
above are correct?
• The report is an updated analysis
of the 2022 Tiger Census. • a. Only one
• This report is based on further • b. Only two
analysis of the same data that was • c. All Three
used to release 2022 tiger census. • d. None

om
• India is home to 75% of the
world’s tiger population with an Answer : b
estimated 3,682 of the big cats in Management Effectiveness

.c
the wild. Evaluation (MEE)
• Among the country’s 53 tiger Recently Management Effectiveness
preserves, Jim Corbett Tiger
Reserve in Uttarakhand has
ap
Evaluation of Tiger Reserves: Final
Report- 5th cycle was published by
maximum number of tigers. National Tiger Conservation Authority
m
• Central India and the Shivalik
• MEE is defined as the assessment
Hills and Gangetic Plains
m

of how well protected areas


witnessed a notable increases in
(PAs) are being managed,
xa

tiger population, particularly in


primarily, whether they are
the states of Madhya Pradesh,
protecting their values and
Uttarakhand, and Maharashtra.
.e

achieving the goals and


• The population of tigers has
objectives agreed upon.
increased by 49% in Madhya
w

• MEE is the framework of


Pradesh, which is the highest in
the World Commission on
w

India, despite recording


Protected Areas of International
the highest tiger mortality.
W

Union for Conservation of Nature


and Natural Resources (IUCN).
• India is the only nation in the
128) Consider the following statements world to have institutionalised
with respect to Management and effectively completed five
Effectiveness Evaluation (MEE) cycles of MEE of Tiger Reserves
in the country.
1. It is the assessment of the
Protected Areas in the country. 5th cycle of MEE of Tiger Reserves
2. It is a framework of the United
• It is jointly conducted by the
Nations Environment Programme
National Tiger Conservation
(UNEP).
Authority (NTCA) and the
3. India is the only nation in the
Wildlife Institute of India (WII).
world to have effectively

Www.iasbio.com | Www.exammap.com | WhatsApp for UPSC NOTES +918987187161


• India has a network of 53 Tiger • Cell-free DNA (cfDNA) are
Reserves spread across 18 States. some fragments of nucleic
Of these, a total of 51 Tiger acids that are present in the
Reserves have been circulatory system.
independently evaluated through • cfDNA can be generated and
the MEE process. released from a cell in a number
• Results of the fifth cycle of MEE of ways including when a cell is
of Tiger Reserves in 2022 dying and the nucleic acids
indicate an overall mean score of become degraded.
78.01% (ranging between 50% to • The release of cfDNA could

om
94%) for 51 Tiger Reserves. occur together with a variety of
• A total of 12 Tiger Reserves have processes, including those
achieved ‘Excellent’’ category, required for normal development,

.c
followed by 21 Tiger Reserves in those related to the development
‘Very Good’ category, 13 Tiger ap of certain cancers and those
Reserves in ‘Good’ category and associated with several other
5 Tiger Reserves in ‘Fair’ diseases.
m
category. • cfDNA is useful tool to
• Periyar Tiger Reserve achieved understand human diseases and
m

the top score of 94.53. to use the knowledge to improve


diagnosis, monitoring, and
xa

129) Consider the following statements prognosis.


with respect to Cell-free DNA • The genetic abnormality such as
Trisomy 21 (Down's syndrome)
.e

1. Cell-free DNA are fragments of can be screened using Cell-free


nucleic acids that are present in
w

DNA in prenatal testing.


the circulatory system. • Another emerging application of
w

2. Application of cell-free DNA cfDNA is in the early detection,


includes the early detection, diagnosis, and treatment of
W

diagnosis, and treatment of cancers.


cancers.
130) Consider the following statements
Which of the following statement(s) with respect to Structured Financial
is/are correct? Messaging System (SFMS)
• a. 1 only 1. It is a messaging system that
• b. 2 only provides secure financial
• c. Both 1 and 2 messages to financial institutions
• d. Neither 1 nor 2 outside India.
2. The system is developed by the
Answer : c National Payments Corporation
Cell-free DNA of India (NPCI).

Www.iasbio.com | Www.exammap.com | WhatsApp for UPSC NOTES +918987187161


Which of the following statement(s)
is/are correct?

• a. 1 only
• b. 2 only
• c. Both 1 and 2
• d. Neither 1 nor 2

Answer : d
Structured Financial Messaging

om
System (SFMS)

• SFMS is a messaging system


that provides secure financial

.c
messages to financial


institutions within India.
SFMS supports ISO 20022 which
ap
is superior to MT (Message Text)
m
standards that is used by SWIFT
(Society for Worldwide
m

Interbank Financial
Telecommunications).
xa

• SFMS provides certain additional


message types for interbank
.e

transactions such as Letters of


Credit which can be used by
w

foreign banks as well.


• India’s SFMS is both cheaper
w

and technically more capable


W

than SWIFT.
• It has the capacity to be
internationalised owing to its
advanced capabilities.
• It is developed by the Indian
Financial Technology & Allied
Services (IFTAS), a wholly-
owned subsidiary of the Reserve
Bank of India (RBI).
• IFTAS designs, deploys &
provides the essential IT-related
services, required by the Reserve
Bank of India, banks, and
financial institutions.

Www.iasbio.com | Www.exammap.com | WhatsApp for UPSC NOTES +918987187161

You might also like